You are on page 1of 40

At what point in an axial-flow turbojet engine will the highest gas pressures occur?

At the compressor outlet.


The highest gas pressure in an axial-flow turbojet engine occurs at the outlet ofthe
compressor. The compressor outlet is the same as the burner inlet.

Identify a function of the nozzle diaphragm in a turbojet engine.


To direct the flow of gases to strike the turbine buckets at a desired angle.
One ofthefunctions ofthe nozzle diaphragm in a turbojet engine is to deflect the
gases to a specific angle in the direction ofthe turbine wheel rotation.

What is the profile of a turbine engine compressor blade?


A cutout that reduces blade tip thickness.
'Profile' in a rotor blade ofa turbine-engine compressor is a reduction in the thickness ofthe
blade tip. Profiles prevent serious damage to the blade or the housing ifthe blade should
contact the compressor housing.

The fan rotational speed of a dual axial compressor forward fan engine is the same as the?
Low-pressure compressor.
Thefan is a portion ofthe low-pressure compressor ofa dual a.xial—flow compressor. The
rotor blades in thefan section are long enough that the air they move passes around the
outside ofthe gas generator portion ofthe engine. Thefan rotational speed is the same as that
ofthe low-pressure compressor.

The abbreviation ‘P’ with subscript t7 used in turbine engine terminology means:
The total pressure at station No. 7.
Pressures in a turbine engine are identified according to their type and to the location at
which they are measured. Pt7 is the total pressure (the pressure a body ofmovingfluid has
when its movement is stopped) measured at station 7, the turbine discharge.

The blending of blades and vanes in a turbine engine


» should be performed parallel to the length of the blade using smooth contours to minimize stress points.
< %v”’-¢.
Blending is a method ofhand recontouring damaged compressor blades and vanes using
smallfiles, emery cloth and honing stones. Blending is done parallel to the length ofthe blade
to minimize stress concentrations and to restore a smooth a shape to the surface.

What turbine engine section provides for proper mixing of the fuel and air?
Combustion section.
In a turbojet engine, compressed airfiom the compressor is directed into the combustion
section. Fuel is sprayedfi"om nozzles in the combustion section where it mixes with the air and
burns.

In a gas turbine engine, combustion occurs at a constant


pressure.
The energy exchange cycle used in a turbojet engine is the Brayton cycle which is a constant-
pressure cycle. The pressure ofthe air remains relatively constant as the energyfrom the
burningfuel is added to it. Since the pressure remains constant as the volume ofthe gas
increases, its velocity increases.

Which statement is true regarding jet engines?


At the higher engine speeds, thrust increases rapidly with small increases in RPM.
The relationship between thrust and RPM is such that the amount ofthrust increases rapidly
as the engine speed increases. A small change in speed at low RPM will not produce nearly as
much increase in thrust as the same amount ofchange at high RPM.

Some high-volume turboprop and turbojet engines are equipped with two-spool or split
compressors. When these engines are operated at high altitudes, the
Iow-pressure rotor will increase in speed as the compressor load decreases in the lower density air.
The high-pressure rotor ofa two-spool (split—spool) compressor is governedfor speed, but
the low~pressure rotor is free to operate at its own best speed. As the air density decreases at
altitude, the compressor load decreases and the low~pressure rotor increases its speed.

TURBINE QUESTIONS 1
Gas turbine engines use a nozzle diaphragm which is located on the upstream side of the turbine
wheel. One of the functions of this unit is to
increase the velocity of the heated gases flowing past this point.
One ofthefunctions ofthe nozzle diaphragm in a turbojet engine is to increase the velocity of
the heated gasesflowing through it. In speeding up this gas, a portion ofthe heat andpressure
energy is turned into velocity energy, which is converted to mechanical energy by the turbine
rotor blades.

Where is the highest gas pressure in a turbojet engine?


In the entrance of the burner section.
The highest gas pressure in a turbojet engine occurs at the outlet ofthe compressor. The
compressor outlet is the same as the entrance ofthe burner section.

An exhaust cone placed aft of the turbine in a jet engine will cause the pressure to
increase and the velocity to decrease.
The exhaust cone on a turbojet engineforms a divergent duct, which increases the pressure of
the exiting gases and decreases their velocity.

What is the function of the stator vane assembly at the discharge end of a typical axial-flow
compressor?
To straighten airflow to eliminate turbulence.
The stator vanes at the discharge ofan axial-flow compressor are called the straightening
vanes. They are used to straighten the airflow to eliminate turbulence.

The turbines near the rear of a jet engine


drive the compressor section.
Thefunction ofthe turbines near the rear end ofa turbojet engine is to drive the compressor.

When starting a turbine engine,


a hot start is indicated if the exhaust gas temperature exceeds specified limits. i<~“°" ’x
A hot start is indicated ifthe engine starts but the exhaust gas temperature exceeds specified
limits.

In the dual axial-flow or twin spool compressor system, the first stage turbine drives the
N2 compressor.
Thefirst-stage turbine in a twin-spool turbojet engine drives the high-pressure compressor.
This is the second stage ofcompression and is called the N2 compressor.

Cracks may occur in hot section components of a turbine engine if they are marked during
inspection with
a lead pencil.
Hany ofthe components in the hot section ofa turbojet engine are marked with a lead pencil,
there is a good chance that the carbonfrom the graphite will infuse the metal and make it
brittle. This will cause the metal to crack.

When starting a turbine engine, a hung start is indicated if the engine


fails to reach idle RPM.
A hung start in a turbine engine is a start in which the engine lights off but is unable to
accelerate to a speed high enough to keep running without help from the starter.

What are the two main sections of a turbine engine for inspection purposes?
Hot and cold.
A gas turbine engine is divided into a hot section and a cold section for inspection purposes.
The cold section includes the engine inlet, the compressor and the dzfluser. The hot section
includes the combustion section, the turbines and the exhaust system.

What are the two basic elements of the turbine section in a turbine engine?
Stator and rotor.

TURBINE QUESTIONS G1 2
The turbine section ofa gas turbine engine consists oftwo basic elements, the stator and the
rotor.

The function of the exhaust cone assembly of a turbine engine is to


straighten and collect the exhaust gases into a solid exhaust jet.
The primaryfiinction ofan exhaust cone assembly used on a turbine engine is to collect the
exhaust gases after they pass through the turbine and convert them into a solid, high-velocity
exhaustjet.

What are the two functional elements in a centrifugal compressor?


Impeller and diffuser. _
A centrifugal compressor used in a gas turbine engine has two basicfunctional elements, the
impeller and the diffuser. The impeller adds energy to the airflowing through the engine and
speeds it up. The dzfluser slows the air down and increases its pressure.

What must be done after the fuel control unit has been replaced on a turbine engine?
Retn'm the engine.
After afiiel control has been replaced on a turbine engine, the engine must be re-trimmed.
Trimming a gas turbine engine consists ofaahusting thefuel control to give the engine the
correct idle RPM and maximum RPM.

What is the most satisfactory method of attaching turbine blades to turbine wheels?
The fir-tree design.
One ofthe most satisfactory methods ofattaching turbine blades to the turbine wheel in a gas
turbine engine is thefir-tree method. The root ofthe turbine blade is shaped like a fir tree, and
itfits loosebl in a matching slot in the turbine wheel. The blades are loose in the wheel when
the wheel is cold, but when the wheel and the blades get hot, they expand and the blades
become tight in the wheel. '

A turbine engine compressor which contains vanes on both sides of the impeller is a
double entry centrifugal compressor. ‘
3%
A centrwigal compressor with vanes on both sides ofthe impeller is called a double-entry
centrifugal compressor.

What is the first engine instrument indication of a successful start of a turbine engine?
A rise in the exhaust gas temperature.
When starting a gas turbine engine, thefirst indication ofa successful start is a sudden rise in
the exhaust-gas temperature.

Turbine discharge pressure is identified in service manuals and by engine instruments as


Pt7.
Turbine discharge pressure is identified on the instruments and in service manuals by the
abbreviation Pt7.

Who establishes the recommended operating time between overhauls (TBO) of a turbine engine
used in general aviation?
The engine manufacturer. , - -
The engine manufacturer establishes the recommended TBO ofa turbine engine, and these
times are approved by the FAA.

The basic gas turbine engine is divided into two main sections: the cold section and the hot
section.
(1) The cold section includes the engine inlet, compressor, and turbine sections.
(2) The hot section includes the combustor, diffuser, and exhaust sections.
Regarding the above statements,
neither No. l nor No. 2 is true.
Statement (I) is not true. The cold section ofa turbine engine does not include the turbine
sections. Statement (2) is not true, because the dzfluser is part ofthe cold section ofthe engine

TURBINE QUESTIONS Q; 3
(1) Gas welding and straightening of turbine engine rotating airfoils does not require special
equipment.
(2) Gas welding and straightening of turbine engine rotating airfoils is quite often recommended
by the manufacturer.
Regarding the above statements,
neither N0. 1 nor No. 2 is true.
Neither statement is true, because welding and straightening ofrotating airfoils in a gas
turbine engine require special equipment. Quite often, neither procedure is authorized by the
engine manufacturer.

A type of repair to turbine engine compressor blade coded areas is accomplished by a procedure
termed blending.
(1) Blending is a hand method of recontouring damaged blades and vanes.
(2) Blending requires the use of small files, emery cloth, and honing stones.
Regarding the above statements,
both No. 1 and No. 2 are true.
Both statements are true, because blending is a hand method ofrecontouring damaged
turbine-engine compressor blades and vanes by using smallfiles, emery cloth and honing
stones.

Who establishes the recommended operating time between overhauls (TBO) of a turbine engine
used in air carrier operation?
The engine manufacturer.
Recommended engine overhaulperiods are established by the engine manufacturer and are
included in the FAA-approved inspection program used by the air carrier.

Who establishes mandatory replacement times for critical components of turbine engines?
The engine manufacturer.
The Instructionsfor Continued Airworthiness, prepared by the engine manufacturer and
approved by the FAA, contain the mandatory replacement timesfor critical components ofthe
engines to which the instructions apply. _<‘“*>.

Main bearing oil seals used with turbine engines are usually what type(s)?
Labyrinth and/or carbon rubbing.
The bearing housing ofa turbine engine usually contains seals to prevent loss ofoil into the
gas path. Oil seal are usually ofthe labyrinth or carbon rubbing type.

How does a dual axial-flow compressor improve the efficiency of a turbojet engine?
Higher compression ratios can be obtained.
The use oftwo axial-flow compressors turning at dzfiierent speeds allows higher compression
ratios to be obtained without the danger ofcompressor stall.

Three types of turbine blades are


impulse, reaction, and impulse-reaction.
Three basic types ofturbine blades used in gas turbine engines are impulse, reaction and
impulse-reaction.

A turboprop powerplant propeller


accounts for 75 to 85 percent of the total thrust output.
A turboprop engine produces its thrust indirectly since the turbinefurnishes torque to the
propeller, which in turn produces the thrust. The propeller accountsfor about 75% to 85% of
the total thrust output.

An advantage of the axial-flow compressor is its


high peak efficiency.
An axial—flow compressor has an advantage over a centrifugakflow compressor in that it has
a higherpeak efiiciency.

What is the purpose of the stator blades in the compressor section of a turbine engine?
Control direction of the airflow.

TURBINE QUESTIONS 4
The stator blades in an axial-flow compressor convert the high velocity ofthe air into
pressure. They also direct the airflow to each ofthe rotor stages to obtain the maximum
possible blade efliciency.

What is the purpose of the diffuser section in a turbine engine?


To increase pressure and reduce velocity.
The difiiiser section in a centrifugal-flow turbojet engine reduces the velocity ofthe air as it
leaves the compressor and increases its pressure.

Where do stress rupture cracks usually appear on turbine blades of turbojet engines?
Across the leading or trailing edge at a right angle to the edge length.
Stress rupture cracks usually appear as tiny hairline cracks on or across the leading or
trailing edge ofthe turbine blades at right angles to the edge length ofthe blade.

In which type of turbine engine combustion chamber is the case and liner removed and installed
as one unit during routine maintenance?
Can.
The can-type combustion chamber ofa turbojet engine consists ofan outer case, or housing
and inside it is a perforated stainless steel combustion chamber or liner. The case and liner
are removed as a unitfor routine maintenance.

The diffuser section of a jet aircraft engine is located between


the compressor section and the burner section.
The dzfiuser is an annular chamberfitted with a number ofvanes thatform a series of
divergentpassages between the centrifugal compressor and the burner section. As the difluser
vanes direct theflow ofair into the burners, they increase the pressure ofthe air and decrease
» its velocity.

Which of the following are the most common types of thrust reversers used on turbine-engine-
powered aircraft?
.a%g Mechanical blockage and aerodynamic blockage.
Mechanical blockage and aerodynamic blockage thrust reversers are the most commonly used
types. Mechanical blockage is accomplished by placing a movable obstruction in the exhaust-
gas stream, usually to the rear ofthe nozzle. The exhaust gases are mechanically blocked and
diverted at a suitable angle in the reverse direction by an inverted cone. In the aerodynamic
blockage~type thrust reverser, thin airfoils are placed in the gas stream immediately after the
exhaust nozzle to deflect the exhaust gasesforward.

When the leading edge of a first-stage turbine blade is found to have stressrupture cracks, which
of the following should be suspected?
Overtemperature condition.
Stress rupture cracks or deformation ofthe leading edge ofthefirst-stage turbine blades are
usually caused by an overtemperature condition. Overtemperature operation must be
suspected when finding this type ofdamage. J

Damage to turbine vanes is apt to be greater than damage to compressor vanes because turbine
vanes are subjected to much greater
heat stress.
Turbine vanes operate under extremes oftemperature, and they are therefore much more
likely to be damaged by heat stresses than compressor vanes, which are located in the cold
section ofthe engine.

Which of the following is the ultimate limiting factor of turbine engine operation?
Turbine inlet temperature.
The turbine-inlet temperature (TH) is the ultimate limitingfactor in the operation ofa
turbojet engine. The temperature in a turbine engine is the highest at the inlet ofthe turbine.

How is the turbine shaft usually joined to the compressor rotor of a centrifugal compressor
turbine engine? p
Splined coupling.

TURBINE QUESTIONS Q 5
Theforward end ofthe turbine shaft is splined, and this splinefits into a coupling device
between the compressor and the turbine shafi‘.

Which of the following engine variables is the most critical during turbine engine operation?
Turbine inlet temperature.
Turbine-inlet temperature B the highest temperature inside a turbine engine. Therefore, it is
the most critical variable ofengine operation. It is impractical to measure turbine—inlet
temperature in most engines. So, the temperature-measuring thermocouples are usually
installed at the turbine discharge. The turbine-outlet temperature gives a relative indication of
the temperature at the turbine inlet. Therefore, ifthe turbine-outlet temperatures are kept
within range, it can be assumed that the turbine-inlet temperatures are also within range.

Reduced blade vibration and improved airflow characteristics in gas turbines are brought about
by
shrouded turbine rotor blades.
Shrouded turbine blades are used to reduce blade vibration and improve the airflow
characteristics through the turbine.

Which turbine engine compressor offers the greatest advantages for both starting flexibility and
improved high-altitude performance?
Split~spool, axial-flow.
The two-spool (split-spool) axial-flow compressor ofiers the greatest startingflexibility and
improved high~altitude performance ofany ofthe gas turbine engine configurations.

Jet engine turbine blades removed for detailed inspection must be re-installed in
the same slot.
Turbine blades are individually weighed and codedfor installation in the disks in such a way
that they best distribute the weight evenly around the disk. Ifa blade is removedfrom a disk
for inspection, it must be reinstalled in the same slotfrom which it was removed.

An advantage of the centrifugal-flow compressor is its high


pressure n'se per stage.
A centrifugal compressor is simple and rugged and it can be made at a relatively low cost.
The pressure rise, which is produced by expansion ofthe gas in the dzfluser manifold and by
the conversion ofkinetic energy ofmotion into static pressure, is high for each stage.

The highest heat-to-metal contact in a jet engine is the


turbine inlet guide vanes.
The highest heat-to-metal contact inside ajet engine occurs at the entrance to thefirst stage
ofthe turbine. Ihis is at the turbine-inlet guide vanes.

Which two elements make up the axial-flow compressor assembly?


Rotor and stator.
An axial-flow compressor is made up ofrotors (the rotating part ofthe compressor) and
stators (the stationary part ofthe compressor).

The two types of centrifugal compressor impellers are


single entry and double entry.
The two types ofcentrifitgal compressors used in turbojet engines are single-entry and
double-entry. A single-entry compressor has scrolls on only one side, while a double-entry
compressor has scrolls on both sides. A double-entiy compressor is much like two single-entry
compressors back to back.

Between each row of rotating blades in a turbine engine compressor, there is a row of stationary
blades which act to diffuse the air. These stationary blades are called
stators.
The stationary blades between each set of rotating blades in an axial-flow turbine-engine
compressor are called stators. Thefitnction ofthe stators is to receive the airfrom each stage
ofthe compressor and deliver it to the next stage at the proper velocity, direction, and
pressure.

TURBINE QUESTIONS Q 6
Standard sea level pressure is
29.92 inches Hg.
Standard sea-level atmospheric pressure is 29.92 inches ofmercury (inches Hg), 760
millimeters ofmercury (mm Hg), 1013.2 millibars (mb), or I 4. 69 pounds per square inch
CW)-
Using standard atmospheric conditions, the standard sea level temperature is
59°F.
The standard sea-level temperaturefor computing the power ofa gas turbine engine is 15°
Celsius, or 59°Fahrenheit.

When aircraft turbine blades are subjected to excessive temperatures, what type of failures
would you expect?
Stress rupture.
A turbine that has been subjected to excessive temperatures is likely to have blades that show
indications ofstress-rupturefailure. Stress-rupture cracks appear as minute hairline cracks
on or across the leading edge ofthe blade at right angles to the edge.

In an axial-flow compressor, one purpose of the stator vanes at the discharge end of the
compressor» is to
straighten the airflow and eliminate turbulence.
The stator vanes located at the discharge end ofan axial-flow compressor are used to
straighten the airflow and eliminate turbulence ofthe air as it enters the combustors.

Compressor field cleaning on turbine engines is performed primarily in order to


prevent engine performance degradation, increased fuel costs, and damage or corrosion to gas path
surfaces.
Foreign deposits on the compressor rotor and stator vanes reduce aerodynamic efficiency of
the blades, degrade the engine performance, and increasefuel costs. Compressorfield
cleaning removes salt or dirt depositsfrom the blades and vanes, restores efliciency, and
prevents corrosion ofthe surfaces along the gas path.

The two types of compressors most commonly used in jet engines are
centrifugal and axial.
The two types ofcompressors used in turbojet engines are centrifugal compressors and axial-
flow compressors.

A purpose of the shrouds on the turbine blades of an ax;ial~flow engine is to


reduce vibration.
Some turbine blades have shrouds on their outer ends that contact the shroud on the adjacent
blade. These shroudsform a band around the outer perimeter ofthe wheel that improves the
efiiciency ofthe wheel and reduces the vibration ofthe blades.

In a dual axial-flow compressor, the first stage turbine drives


N2 compressor.
Thefirst-stage turbine in a twin-spool turbojet engine drives the high-pressure compressor,
the N2 compressor.

What should be done if a turbine engine catches fire during starting?


Turn off the fuel and continue cranking.
Ea turbine engine catchesfire in the process ofstarting, turn thefitel ofi’and continue
cranking the engine toforce enough air through the engine to blow thefire out.

What is the proper starting sequence for a turbojet engine?


Starter, ignition, fuel.
The proper sequence for starting a turbojet engine is to engage the starter to start the
compressor turning, then turn on the ignition andfinally, turn on thefitel.

In~flight turbine engine flameouts are usually caused by

TURBINE QUESTIONS Q 7
interruption of the inlet airflow.
In-flight turbine-engineflameout is usually caused by an interruption ofthe inlet airflow.
Turbulence, ingestion ofbirds or ice, orflight manoeuvres that create such a high angle of
attack that the airflow through the engine is interrupted, will change thefuel-air mixture in
the engine, and the engine is likely to flame out.

What is used in turbine engines to aid in stabilization of compressor airflow during low thrust
engine operation?
Variable guide vanes and/or compressor bleed valves.
Some axial;flow gas turbine engines use variable inlet guide vanes and bleed-air valves to
stabilize the airflow through the compressor during low-thrust operations. Ifthe compressor
RPM is high, relative to the amount ofairflowing through the engine, the angle ofattack of
the compressor blades will become excessive and a compressor surge or stall can develop. T0
prevent a compressor stall, the inlet guide vanes are turned to the correct angle, and the
bleed-air valves are automatically opened by actuators controlled by thefil€l control. The
bleed~air valves reduce the backpressure and allow more air toflow through the compressor
to reduce the angle ofattack ofthe compressor blades.

In a turbine engine with a dual-spool compressor, the low speed compressor


seeks its own best operating speed.
The high-pressure rotor ofa two-spool (split-spool) compressor is governedfor speed, but the
low—pressure rotor isfree to operate at its own best speed. As the air density decreases at
altitude, the compressor load decreases and the low-pressure rotor increases its speed.

What is the function of the inlet guide vane assembly on an axial-flow compressor?
Directs the air into the first stage rotor blades at the proper angle.
Inlet guide vanes used with an axial-flow compressor change the angle ofairflow to direct it
into thefirst stage rotor blades at the proper angle. The inlet guide vanes do not change either
the velocity or the pressure ofthe inlet air.

-
Hot spots on the tail cone of a turbine engine are possible indicators of a malfunctioning fuel i .
nozzle or
a faulty combustion chamber.
Hot spots, which are localized areas ofoverheating in the tail cone ofa gas turbine engine,
are usually caused by a malfunctioningfuel nozzle or afaulty combustion chamber that
prevent a uniform flow ofcooling air.

The stator vanes in an axial-flow compressor


convert velocity energy into pressure energy.
Stator vanes in an axial-flow compressor convert velocity energy that has been put into the air
by the rotors into pressure energy. The stator vanes also direct the air into thefollowing stage
ofrotor blades in the correct direction.

What happens to velocity as subsonic air flows through a convergent nozzle?


Increases.
When subsonic airflows through a convergent nozzle, its velocity increases and its pressure
decreases. This action is in accordance with Bernoulli ’s principle, which gives us the
relationship between kinetic energy andpotential energy ina column ofmovingfluid. The
kinetic energy relates to the velocity ofthefluid and the potential energy relates to the
pressure ofthefluid.

What happens to velocity as supersonic air flows through a divergent nozzle?


Increases.
When supersonic airflows through a divergent nozzle (a nozzle whose cross-sectional area
increases in the direction ofairflow), its velocity increases and its pressure decreases.

What happens to pressure as subsonic air flows through a convergent nozzle?


Decreases.
When subsonic airflows through a convergent nozzle, its velocity increases and its pressure
decreases. This action is in accordance with Bernoulli's principle, which gives us the

TUF-ZBlNE QUESTIONS Q‘ a
relationship between kinetic energy andpotential energy in a column ofmovingfluid. The
kinetic energy relates to the velocity ofthefluid and the potential energy relates to the
pressure ofthefluid.

What happens to pressure as supersonic air flows through a divergent nozzle?


Decreases.
When supersonic airflows through a divergent nozzle (a nozzle whose cross-sectional area
increases in the direction ofairflow), its velocity increases and its pressure decreases.

Anti-icing of turbojet engine air inlets is accomplished by


engine bleed air ducted through the critical areas.
Anti-icing Qureventing theformation ofice) is accomplished on the air inlets ofa turbojet
engine by ducting hot engine bleed-air through the areas on which the ice is lilcehi to form.

Generally, when starting a turbine engine, the starter should be disengaged


after the engine has reached self-accelerating speed.
When starting a turbojet engine, the starter is left engaged until the engine reaches its self
accelerating speed. Ifthe starter is disengaged too soon, the engine mayfail to accelerate to
its idle RPM, and a ’hung start’ results.

What is the primary advantage of an axial-flow compressor over a centrifugal compressor?


Greater pressure ratio.
The primary advantage ofan axial-flow compressor over a centrmtgal compressor is that
axial-flow compressors are capable ofproducing higher pressure ratios and they have
relatively high efliciencies.

What is the purpose of blow-in doors in the induction system of a turbine engine aircraft?
Admit air to the engine compartment during ground operation when the engine air requirements are in
excess of the amount the nonnal intake system can supply.
When a turbine-engine-powered aircraftproduces high power with little or no ram airflowing
into the inlet duct, there is a possibility that there will not be enough airflowing through the
‘sees?’
engine to prevent compressor stall. Spring-Loaded blow-in doors located around the
periphery ofthe inlet duct will be opened by the low pressure at the inlet and will admit the
needed air to the compressor. Ram airflowing into the inlet duct increases the pressure at the
compressor inlet and allows the springs to close the blow-in doors.

What is meant by a double entry centrifugal compressor?


A compressor with vanes on both sides of the impeller.
A double-entry centrifugal compressor is a compressor that has vanes on both sides ofthe
impeller. The intake air is ducted into the impeller at both itsfront and back sides. The
accelerated air is takenfiom its rim.

What is the major function of the turbine assembly in a turbojet engine?


Supplies the power to turn the compressor.
The majorfunction ofthe turbine assembly in a turbojet engine is that ofsupplyingpower to
turn the compressor.

Stator blades in the compressor section of an axial-flow turbine engine


decrease the air velocity and prevent swirling.
Stator vanes in an axial-flow compressor convert velocity energy that has been put into the air
by the rotors into pressure energy. They slow the air, which increases its pressure. The stator
vanes also direct the air into thefollowing stage ofrotor blades in the correct direction.

A gas turbine engine comprises which three main sections?


Compressor, combustion, and turbine.
The three main sections ofa turbine engine are: the compressor, which increases the
pressure of the air entering the engine; the combustors, in which energyfiom burningfuel is
added to the compressed air; and the turbine, which drives the compressor. _

What type of turbine blade is most commonly used in aircraft jet engines‘?

TURBINE QUESTlONS Q1 9
Reaction-impulse (or impulse-reaction).
The most common type ofturbine blades used in modern aircraftjet engines is the reaction-
impulse type. A reaction-impulse turbine blade has an impulse section at its root and a
reaction section at its tip. The exitpressure ofa reaction-impulse turbine blade is relatively
constant across its length.

What is the primary factor which controls the pressure ratio of an axial-flow compressor?
Number of stages in compressor.
The pressure ratio ofan axial-flow compressor is ajunction ofthe number ofstages of
compression (the number ofstages ofcompressor rotors and stators).

(1) A turbine engine axial-flow compressor is made up of a series of rotating airfoils called rotor
blades and a stationary set of airfoils called stator vanes.
(2) In a turbine engine, a row of rotating and stationary blades is called a stage.
Regarding the above statements,
both No. 1 and No. 2 are true.
Both statements are true. An axialflow compressor is made up ofa series ofrotating airfoils,
called rotor blades and stationary airfoils, called stator vanes. Each consecutive pair ofrotor
blades and stator vanes constitutes a pressure stage.

(1) In a turbine engine axial-flow compressor, each consecutive pair of rotor and stator blades
constitutes a pressure stage.
(2) In a turbine engine axial-flow compressor, the number of rows of stages is determined by the
amount of air and total pressure rise required.
Regarding the above statements,
both No. l and No. 2 are true.
Both statements are true. Each consecutive pair ofrotor blades and stator vanes in an axial-
flow compressor constitutes a pressure stage. The number ofpressure stages in the
compressor is determined by the amount ofair and the total pressure rise required by the
engine.

The air passing through the combustion chamber of a jet engine is


used to support combustion and to cool the engine.
Part ofthe airflowing through the combustion chamber ofajet engine mixes with thefuelfor
combustion. The majority ofthe air, however, passes between the outer casing and the liner
and is used to cool the combustion gases.

The stators in the turbine section of a gas turbine engine


increase the velocity of the air. '
The turbine in a gas turbine engine extracts energyfiom the burning gases as they pass
through it. The stators in front ofthe rotating turbine wheels increase the velocity ofthe gases
and direct them so they will strike the rotors at the correct angle.

The procedure for removing the accumulation of dirt deposits on compressor blades is called
field cleaning.
Field cleaning ofa turbine engine is the removal ofcontaminantsfrom the blades ofthe
compressor by running wash water or an abrasive through the engine.

Which of the following is used to accomplish internal inspection of installed turbine engines?
1. Infrared photography.
2. Ultrasound.
3. A borescope.
4. Fluorescent penetrant and ultraviolet light.
3.
The inspection ofthe interior ofa turbine engine installed on an aircrafi can be performed by
the use ofa borescope.

What is the possible cause when a turbojet engine indicates no change in power setting
parameters, but oil temperature is high?
Unusual scavenge pump oil flow.

TURB!NE QUESTlONS 10
Ifthere is an increase in the lubricating~oil temperature without any change in the power-
setting parameters in the engine, it could indicate a problem with the scavenge pump oilflow.
The oil temperature is controlled by routing hot oil through thefuel-oil heat exchanger, and
allowing cold oil to bypass the heat exchanger. ifhot oil bypasses the heat exchanger, its
temperature will go up.

Which of the following is NOT a factor in the operation of an automatic fuel control unit used on
turboj et engines?
Mixture control position.
The automaticfuel-control unit used on turbojet engines senses all thefactors listed in these
alternatives except the mixture-controlposition. A turbojet engine does not have a mixture
control.

Newton's First Law of Motion, generally termed the Law of Inertia, states:
Every body persists in its state ofrest, or of motion in a straight line, unless acted upon by some outside
force.
Newton's First Law ofMotion tells us that every body at rest will try to remain at rest and
every body in motion will try to remain in motion in a straight line, at the same speed, unless
it is acted upon by an outsideforce

A turbine engine hot section is particularly susceptible to which kind of damage?


Cracking.
Cracking is one ofthe most widelyfoundforms ofdamage in the hot section ofa gas turbine
engine. Vibration and the extremes oftemperature cause the thin metal ofwhich hot-section
components are made, to crack.

Dirt particles in the air being introduced into the compressor of a turbine engine will form a
coating on all but which of the following?
Turbine blades.
Dirtparticles introduced into the turbine engine will cause a coating to form on the casings,
inlet guide vanes and compressor blades. The extreme heat in the turbine section prevents the
coatingfrom forming on the turbine blades.

Severe rubbing of turbine engine compressor blades will usually cause


gelling.
Severe rubbing ofthe turbine-engine compressor blades will usually result in galling, which is
a transfer ofmetalfiom one surface to another. _

Which of the following influences the operation of an automatic fuel control unit on a turbojet
engine? ~
Burner pressure.
Most automaticfuel control unitsfor turbojet engines sense inlet air temperature, compressor
RPM, burner pressure (compressor discharge pressure) and the position ofthe power lever
(throttle).

If a turbine engine is unable to reach takeoff EPR before its EGT limit is
reached, this is an indication that the
compressor may be contaminated or damaged.
When a turbine engine compressor is contaminated or damaged, the airflow is disturbed, and
the EGT limit may be reached before talceofi’EPR is attained. This condition may be corrected
byfield cleaning the compressor or retrimming thefuel control.

The Brayton cycle is known as the constant


pressure cycle.
The Brayton cycle ofenergy release used in a gas turbine engine is known as the constant-
pressure cycle. Energy added to the airflowing through the engine by the burningfitel, causes
the volume ofthe air to increase, but the pressure remains relatively constant.

Where is water injected into a turbojet engine for cooling purposes?


Compressor air inlet or diffuser.

TURBINE QUESTIONS Q} 11
Water injected at the compressor air-inletsection or at the dzfluser case ofa turbojet engine
will reduce the hot-section temperatures and allow thefuelflow to be increased. The
increasedfuelflow allows a greater thrust to be developed by the engine.

Continued and/or excessive heat and centrifugal force on turbine engine rotor blades is likely to
cause V
creep.
Creep, which is a permanent elongation ofthe turbine blades caused by heat loads and
centrifugal loads, is likely to occur when the engine has been exposed to continued and/or
excessive heat.

If the RPM of an axial~tlow compressor remains constant, the angle of attack of the rotor blades
can he changed by
changing the velocity of the airflow.
Twofactors aflecting the angle ofattack ofan axial-flow compressor blade are the velocity of
the air through the engine and the RPM ofthe compressor. ifthe airflow entering the engine
is restricted, reducing its velocity, the angle ofattack ofthe compressor blade will increase to
such a point that compressor stall can occur.

The compression ratio of an axial-flow compressor is a function of the


number of compressor stages.
The compressiomratio ofan axial-flow compressor is determined by the number ofstages of
compression.

Which of the following variables affect the inlet air density of a turbine engine?
1. Speed of the aircraft.
2. Compression ratio.
3. Turbine inlet temperature.
4. Altitude of the aircraft.
5. Ambient temperature.
6. Turbine and compressor efficiency.
1, 4, 5. -
Threefactors that afi"ect the density ofthe air taken into the inlet air system ofa turbojet
engine are the speed ofthe aircraft, the altitude at which the aircraft is flying and the ambient
(surrounding) air temperature.

Which of the following factors affect the thermal efficiency of a turbine engine?
1. Turbine inlet temps rature.
2. Compression ratio.
3. Ambient temperature.
4. Speed of the aircraft.
5. Turbine and compressor efficiency.
6. Altitude of the aircraft.
1, 2, 5.
Threefactors that aflect the thermal efiiciency ofa turbine engine are the turbine-inlet
temperature, the compression ratio ofthe compressor and the turbine and compressor
efliciency.

Why do some turbine engines have more than one turbine wheel attached to a single shaft?
To extract more power from the exhaust gases than a single Wheel can absorb.
Some turbine engines have more than one turbine wheel on a single shaft in order to extract
more powerfiom the exhaust gases than a single turbine wheel can absorb.

The exhaust section of a turbine engine is designed to


impart a high exit velocity to the exhaust gases.
The exhaust section ofa turbojet engine is designed in such a way that it gives a high velocity
to the exhaust gases leaving the engine.

Which of the following types of combustion sections are used in aircraft turbine engines?
Multiple-can, annular, and can-annular.

TURBINE QUESTIONS Ca 12
Three basic types ofcombustion sections used in gas turbine engines are the multiple-can
type, the annular type, and the can-annular type.

Why does a turbine engine require a cool-off period before shutting it down?
To allow the turbine Wheel to cool before the case contracts around it.
A rule ofthumbfor turbine engine operation: when an engine has been operated above
approximately 85% RPMforperiods longer than one minute; during the lastfive minutes
before shutdown, the engine should be operated below 85% RPM (preferably at idle) for a
period offive minutes. This prevents the possibility ofthe engine case contracting around the
turbine wheels before they have cooled and contracted to their normal size. It is also
important that all surfaces contacted with engine oil be cooled to their normal operating
temperature, to prevent oil lefi on a hot surfacefrom coking (turning into a hard carbon
deposit).

How many igniters are normally used on a turbine engine having nine burner cans?
Two.
A gas turbine engine using nine separate burner cans, normally has two igniters located in
cans on opposite sides ofthe engine. Interconnect tubesjoin the cans so fire in one ofthe cans
can carry over and ignite thefuel-air mixture in the other cans.

What is meant by a shrouded turbine?


The turbine blades are shaped so that their ends form a band or shroud.
A shrouded turbine is one in which each ofthe blades is made in the shape ofthe letter T.
Each bar on the end ofthe blades touches the other toform a band, or shroud, around the
turbine wheel. The shrouds increase the efliciency ofthe turbine and improve the vibration
characteristics ofthe turbine blades.

What term is used to describe a permanent and cumulative deformation of theturbine blades of a
turbojet engine?
Creep.
Creep zls the stretching (elongation) ofa turbine blade caused by prolonged exposure to high
temperatures and centrifitgalforce.

What is the purpose of the pressurization and dump valve used on turbojet engines?
Allows fuel pressurization of the engine when starting and operating and dumps fuel pressure at engine
shutdown.
Thefuelpressurization and dump valve, as is used on the Pratt & Whitney JT3 engine, serves
two basicfunctions: it pressurizes thefuel during engine start and operation as is neededfor
atomization at thefuel nozzles; and at engine shutdown, itprovides a dump system that
connects thefitel manifolds to an overboard drain.

At what stage in a turbojet engine are pressures the greatest?


Compressor outlet.
The gas pressure inside a turbojet engine is the greatest at the compressor outlet.

In what section of a turbojet engine is the jet nozzle located?


Exhaust.
The rear opening ofa turbine~engine exhaust duct is called the exhaust nozzle (jet nozzle). The
nozzle acts as an orifice, the size ofwhich determines the density and velocity ofthe gases as
they leave the engine.

When a turbojet engine is removed for maintenance or test cell operation, it should be
accomplished ’
in accordance with the manufacturer's instmcfions.
Always refer to the applicable manufacturer's instructions before performing any
phase ofengine removal or installation.

(1) Accumulation of contaminates in the compressor of a turbojet engine reduces aerodynamic


efficiency of the blades.

TURBINE QUESTIONS ('3\ 13


(2) Two common methods for removing dirt deposits from turbojet engine compressor blades are
a fluid wash and an abrasive grit blast.
Regarding the above statements,
both No. l and No. 2 are true.
Both statements are true. Accumulation ofcontaminants in the compressor reduces
aerodynamic efliciency ofthe blades and reduces engine performance. Two common methods
for removing dirt deposits are afluid wash and an abrasive grit blast.

Hot spots in the combustion section of a turbojet engine are possible indicators of
malfunctioning fuel nozzles.
Hot spots are possible indicators ofa serious condition, such as malfunctioningfuel nozzles
or otherfuel-system malfunctions.

Which of the following can cause fan blade shingling in a turbofan engine?
1. Engine overspeed.
2. Engine overtemperature.
3. Large, rapid throttle movements.
4. FOD.
1, 4.
Fan blade shingling is a condition ofthefan in a turbofan engine when the midspan shrouds
on thefan blades overlap in much the same way shingles on a roofoverlap. Fan blade
shingling is caused by the rotatingfan encountering opposition such as engine stall, bird
strike, foreign object damage (FOD), or by engine overspeed.

Compressor stall is caused by


a high angle of attack airflow through the first stages of compression.
A compressor stalls when the angle ofattack ofthe blades becomes excessive. Two factors
afiecting the angle ofattack ofan axial-flow compressor blade are the velocity ofthe air
through the engine and the RPM ofthe compressor. Kthe airflow entering the engine is
restricted, reducing its velocity, the angle ofattack ofthe compressor blade will increase to
such a point that compressor stall can occur. /a<»»t,>‘

A condition known as ‘hot streaking‘ in turbine engines is caused by


a partially clogged fuel nozzle.
Hot streaking is a hot-section condition in which theflame penetrates‘ through the entire
turbine system to the tailpipe. Hot streaking is caused by a partially cloggedfuel nozzle
which does not atomize thefitel into a cone~shapedpattern but rather, allows a smallfitel
stream to flow M ith sufficientforce to cut through the cooling air blanket and impinge directly
on the turbine surfaces.

Which of the following is used to monitor the mechanical integrity of the turbines, as well as to
check engine operating conditions of a turbine engine?
Exhaust gas temperature.
Temperature is one ofthe most important considerations in the operation ofa turbine engine.
Because ofthis, the exhaust-gas temperature, which gives us an indication ofthe turbine-inlet
temperature, allows us to monitor the mechanical integrity ofthe turbines as well as to check
the operating conditions ofthe engine.

The breaking loose of small pieces of metal from coated surfaces, usually caused by defective
plating or excessive loads, is called
flaking.
Bearingflaking causes smallpieces ofmetal to break awayfrom the hardened surface ofthe
bearing race or roller. Thisflaking may be caused by excessive bearing loads or defective
plating.

A severe condition of chafing or fretting in which a transfer of metal from one part to another
occurs is called
gelling.
Galling is a severe condition ofchafing orfietting that occurs when two pieces ofmetal rub
together and transfer metalfrom one piece to the other.

TURBlNE QUESTIONS G 14
Indentations on bearing races caused by high static loads are known as
brinelling.
Brinelling is a condition that occurs in a bearing when the race is indented by the balls or
rollers. Brinelling is caused by high static loads being imposed on the bearings.

Which of the following conditions is usually not acceptable to any extent in turbine blades?
Cracks.
Cracks in a turbine blade is causefor rejection ofthe blade.

(1) Serviceability limits for turbine blades are much more stringent than are those for turbine
nozzle vanes.
(2) A limited number of small nicks and dents can usually be permitted in any area of a turbine
blade.
Regarding the above statements,
only No. l is true.
Statement (1) is true. Serviceability limitsfor turbine blades are much more stringent than
thosefor nozzle vanes. This is particularly truefor blades in thefirst stage because ofthe high
temperature involved. The high centrifugal stresses to which turbine blades are subjected,
require that the blades befree ofcracks in any area and no nicks or dents are allowed to exist
in the root area. Statement (2) is not true. A limited number ofsmall nicks and dents can be
permitted, but only in the areas ofthe blade awayfrom the root area. t

Motor driven impeller and turbine fuel flow transmitters are designed to transmit data
using aircraft electrical system power.
The mass-flowfiielflowmeter using a motor-driven impeller and turbine, transmit their data
using 115-volt AC electrical powerfrom the aircraft electrical system.

In addition to fuel quantity, a computerized fuel system (CFS) with a totalizer-indicator provides
indication of how many of the following?
(,<<*%\>
1. Fuel flow rate.
2. Fuel used since reset or initial start-up.
3. Fuel time remaining at current power setting.
4. Fuel temperature.
Three.
The Computerized Fuel System (CFS) provides the pilot with fuelflow information in pounds
per hour or gallons per hour, gallons or pounds remaining, time remainingforflight at the
current power setting, and gallons usedfrom the initial engine start-up.

The fuel-flow indication data sent from motor driven impeller and turbine fuel flow transmitters
is a measure of
fuel mass-flow.
Thefuelflow indication data sentfrom a motor-driven impeller and turbinefil€lflow
transmitter is a measure ofmassfuelflow. This type offirelflowmeter takes into consideration
both the volume and density ofthefuel.

Which of the following type of electric motors are commonly used in electric tachometers?
Synchronous motors.
Electric tachometers use a three-phase permanent-magnet generator turned by the engine or
helicopter transmission. Tachometer generators produce three—phase alternating current
whoseflequency is determined by the speed the permanent magnet rotor is turned. Inside the
instrument case there is a synchronous motor that spins at the same speed as the magnet in
the generator. A magnetic-drag-indicating system converts the rotational speed ofthe
synchronous motor into angular deflection ofthe tachometerpointer.

Basically, the indicator of a tachometer system is responsive to change in


fiequency.
This question refers to the three-phase AC tachometer generator and synchronous motor
system. An AC tachometer generator produces a voltage whosefrequency is proportional to
the engine speed. The variablefrequency alternating current produced by the tachometer

TURBINE QUESTIGNS L31 15


generator drives a synchronous motor inside the instrument case. The pointer ofa magnetic
drag tachometer is driven by the synchronous motor. It moves across the indicator dial to
indicate the speed ofthe generator rotor.

Which statement is correct concerning a thermocouple-type temperature indicating instrument


system?
It requires no external power source.
A thermocouple-type temperature—indicating system is self-contained and requires no external
power. The voltage generated by a thermocouple is determined by the diflerence in
temperature between the hot (measuring) junction at the cylinder head and the cold
(reference)junction inside the instrument case.

Which of the following is a primary engine instrument?


Tachometer.
ifwe consider the instruments required by the FAA for certificated aircraft as primary’
engine instruments, the tachometer is the only one ofthese instruments that is a primary
engine instrument. .

Engine oil temperature gauges indicate the temperature of the oil


entering the engine.
The temperature indicated on the oil-temperature gauge is the temperature ofthe oil as it
enters the engine. Oil-inlet temperature gives an indication ofthe efiiciency ofthe oil-cooling
system. It lets the pilot know whether or not the oil entering the engine is too hotfor adequate
heat removal.

A common type of electrically operated oil temperature gauge utilizes


either a wheatstone bridge or ratiometer circuit.
The most generally used electrical oil temperature gaugesfor aircraft engines uses either a
Wheatstone bridge or a ratiometer circuit. A temperature probe consisting ofa coil ofthin
wire encased in a stainless steel housing, senses the temperature ofthe oil. The resistance of
the wire changes as the temperature changes. This change ofresistance is indicated on the
instrument as a change in temperature.

(1) Powerplant instrument range markings show whether the current state or powerplant
operation is normal, acceptable for a limited time, or unauthorized.
(2) Powerplant instrument range markings are based on installed engine operating limits which
may not exceed (but are not necessarily equal to) those limits shown on the engine Type
Certificate Data Sheet. -
Regarding the above statements.
both No. 1 and No. 2 are true.
Statement (1) is true. Powetplant instrument range markings show whether the current state
ofpowerplant operation is normal (green arc), acceptablefor a limited time (yellow arc), or
unauthorized (red line). Statement (2) is also true. Powerplant range markings are based on
the installed engine operating limits which arefound in the Aircraft Type Certificate Data
Sheets. These limits may not exceed, but are not necessarily the same as those shown in the
Engine Type Certificate Data Sheets.

Thermocouple leads
are designed for a specific installation and may not be altered.
Thermocouple leads must have a specific resistancefor a given installation, and therefore
they should not be altered.

(1) Engine pressure ratio (EPR) is a ratio of the exhaust gas pressure to the engine inlet air
pressure, and indicates the thrust produced. <
(2) Engine pressure ratio (EPR) is a ratio of the exhaust gas pressure to the engine inlet air
pressure, and indicates volumetric efficiency.
Regarding the above statements,
only No. l is true.
Statement (Z) is true. Engine pressure ratio (EPR) is a ratio ofthe turbine discharge pressure
to the engine inletpressure. It relates to the amount ofthrust the engine is producing.

TURBlNE QUESTIONS Q 16
Statement (2) is not true. The EPR has nothing to do with indicating the volumetric efliciency
ofan engine.

What unit in a tachometer system sends information to the indicator?


The three-phase ac generator.
The tachometer system mentioned in this question uses a three-phase AC generator mounted
on the engine to drive a synchronous motor inside the tachometer indicator.

(1) Generally, when a turbine engine indicates high EGT for a particular EPR (when there is no
significant damage), it means that the engine is out of trim.
(2) Some turbine-powered aircraft use RPM as the primary indicator of thrust produced, others
use EPR as the primary indicator.
Regarding the above statements,
both No. 1 and No. 2 are true.
Statement (1) is true. Ifthe EGT is highfor a particular EPR when there is no significant
. damage, the engine is out oftrim and thefuel control should be aajiusted. Statement (2) is also
true. Turbine engines with centrifugal compressors use RPM as a primary indicator ofthrust
while axial-flow engines use EPR as a primary thrust indicator.

Engine pressure ratio is determined by


dividing turbine outlet total pressure by engine inlet total pressure.
Engine pressure ratio EPR) is the ratio ofthe turbine-discharge pressure to the compressor-
inletpressure. It z's used as a measure ofthe thrust produced by an axial-flow turbine engine.
A ratio isfound by dividing one term by the other. In this instance, the turbine-outlet total
pressure is divided by the engine-inlet (compressor-inlet) total pressure.

Jet engine thermocouples are usually constructed of


chromebalumel.
The thermocouples used to measure EGT or TIT ofa turbojet engine are made ofchrome! and
alumel wire. These temperatures are much higher than the cylinder-head temperature ofa
reciprocating engine that uses thermocouples made ofiron and constantan or copper and
ffi
constantan.

A Bourdon-tube instrument may be used to indicate


1. pressure.
2. temperature.
3. position.
4. quantity.
1 and 2.
A Bourdon tube measures only pressure. But in a Bourdon tube temperature-measuring
instrument, the Bourdon tube is connected by a sealed capillary tube to a bulb containing
methyl chloride. The bulb is placed in the area where the temperature is to be measured. As
the temperature changes, the pressure ofthe methyl chloride changes.

What instrument on a gas turbine engine should be monitored to minimize the possibility of a
‘hot’ start?
Turbine inlet temperature.
Turbine-inlet temperature (TH) or exhaust-gas temperature (EG2) ofa gas turbine engine is
monitored to determine whether the start is normal or ifit is a hot start.

In regard to using a turbine engine oil analysis program, which of the following is NOT true?
Generally, an accurate trend forecast may be made after an engine's first oil sample analysis.
An oil analysis program is a trend indicating system. A sample ofthe oil is taken when the
engine is new and the parts~per-million ofeach ofa number of elements are recorded. As.
subsequent samples aretested on a regular basis the trend ofthe growth ofthe trace elements
are recorded. When the growth ofany element isfar more rapid than it should be, the engine
operator is warned ofpossible impending trouble. Analysis ofthe first sample is only a
starting point and cannot give an accurate forecast alone.

I‘
TURBINE QUESTlONS 5' 17
On a turbine engine, with a fixed power lever position, the application of engine anti-icing will
result in
a decrease in EPR.
ITthe power leverposition is not changed, application ofengine anti-icing will result in a
slight decrease in the EPR indication. Some air is being bledfrom the compressor and it
reduces the totalpressure at the turbine discharge.

Engine pressure ratio is the total pressure ratio between the


front of the compressor and the rear of the turbine.
Engine pressure ratio (EPR) z'.s the ratio ofthe turbine-discharge totalpressure (measured at
the aft end ofthe turbine) to the compressor-inlet totalpressure (measured at thefront end of
the compressor). Total pressure is the pressure a column ofmoving air has when it is stopped.

What would be the possible cause if a gas turbine engine has high exhaust gas temperature, high
fuel flow, and low RPM at all engine power settings?
Turbine damage or loss of turbine efficiency.
A damaged or dirty turbine can cause the engine to have a high exhaust~gas temperature, a
highfuelflow and a low RPM at allpower settings.

What is the primary purpose of the tachometer on an axial-compressor turbine engine?


Monitor engine RPM during starting and to indicate overspeed conditions.
The RPM ofan a.xial—flow gas turbine engine is not used as an indicator ofthrust, but RPM is
important during the engine starting procedure and during high-power operation to prevent
over speeding the engine.

The engine pressure ratio (EPR) indicator is a direct indication of


engine thrust being produced.
In an axial-flow gas turbine engine, the engine pressure ratio (EPR) varies directly with the
thrust the engine is producing.

The exhaust gas temperature (EGT) indicator on a gas turbine engine provides a relative
indication of the
turbine inlet temperature.
The temperature ofthe gases entering the turbine is very difiicult to measure. Because ofthis,
exhaust-gas temperature, which is easy to measure, is used to give an indication ofthe
turbine-inlet temperature.

What instrument indicates the thrust of a gas turbine engine?


Engine pressure ratio indicator.
In an axial-flow gas turbine engine, the engine pressure ratio (EPR) varies directly with the
thrust the engine is producing.

In a turbine engine, where is the turbine discharge pressure indicator sensor located?
Immediately aft of the last turbine stage.
A turbine-discharge pressure-indicator sensor is located immediately aft ofthe last turbine
stage. The turbine-discharge pressure is used with the compressor-inlet pressure tofind the
engine pressure ratio (the EPR).

In What units are turbine engine tachometers calibrated?


Percent of engine RPM.
Gas turbine engine tachometers are calibrated in percent RPM ofthe compressor. It is much
more important that the pilot know whether or not he is operating the engine at its peak speed
or an overspeed condition, than it isfor him to know specific RPMS.

Instruments that provide readings above approximately 10 PSI, such as oil pressure gauges, are
usually what type?
Bourdon tube.
Bourdon tube instruments are normally used to measure such pressures as engine oilpressure
and hydraulic system pressure. »

TURBINE QUESTIONS 18
The RPM indication of a synchronous ac motor-tachometer is governed by the generator
frequency.
The RPM indication ofa synchronous AC tachometer is governed by the generatorfrequency.
The tachometer generator used in this type ofsystem is a permanent magnet generator whose
outputfiequency is determined by the speed it is turned.

A red triangle, dot, or diamond mark on an engine instrument face or glass indicates
the maximum limit for high transients such as starting.
A red triangle, dot, or diamond mark on an engine instrumentface or glass, indicates the
maximum limitfor high transients such as starting.

Which of the following fire detectors are commonly used in the power section of an engine
nacelle?
Rate-of-temperature-rise detectors.
Watch this type ofquestion. The question asksfor the type offire detector that would be used.
Ofthe choices given here, only one is afire detector. The rate-of-temperature-rise detector is
a fire detector. The CO (carbon monoxide) detector, the combustible mixture detector, and
smoke detector are notfire detectors.

What is the function of a fire detection system?


To activate a warning device in the event of a powerplant fire.
A fire detection system activates a warning device in the event ofa powerplantfire, but it does
not discharge thefireextinguishing agent. A switch must be actuated by the pilot orflight
engineer to discharge thefire-extinguishing agent.

How are most aircraft turbine engine fire-extinguishing systems activated?


Electrically discharged cartridges.
Most aircraft-turbine-enginefireextinguishing systems use a high-rate discharge (HRD)
bottle that contains some type ofFreon pressurized with nitrogen. Electrically operated squibs
(explosive charges) rupture the metal seal on the HRD bottle and discharge thefire-
extinguishing agent.

How does carbon dioxide extinguish an engine fire?


By dissipating or displacing oxygen in the immediate area of the fire.
When carbon dioxide is releasedfiom the bottle, it expands and displaces the oxygen in the
immediate area ofthefire. A fire will go out when it is deprived ofoxygen.

What retains the nitrogen charge and fire-extinguishing agent in a high rate of discharge (HRD)
container?
Breakable disk and fusible disk.
The high-rate discharge containers are sealed with a breakable disk that is cut by an
explosive cartridge when thefire pull switch is energized. There is also afilSibl6 disk that will
rupture and release the agent ifthe temperature surrounding the container becomes excessive.
Ifthe bottle is discharged by the normal method, a yellow indicator disk will blow out, but ifit
is discharged by an overtemperature condition, the red indicator disk will blow out.

A continuous-loop fire detector is what type of detector?


Overheat detector.
A continuous—loopfire detection system is aform ofoverheat detector made in theform ofa
loop installed around the engine compartment. It initiates afire warning when afire or
overheat condition changes the electrical characteristics ofthe heat-sensitive material in the
loop.

What is the operating principle of the spot detector sensor in a fire detection system?
A bimetallic thennoswitch that closes when heated to a high temperature.
The spot detector sensor in afire detection system is a bimetallic thermal switch between two
loops ofwire. When a high temperature occurs at the detector, the switch closes and
completes the circuit between the two loops. This initiates thefire warning signal.

TURBINE QUESTIONS GK 19
Which of the following is the safest fire-extinguishing agent to use from a standpoint of toxicity
and corrosion hazards?
Bromotrifluoromethane (Halon 1301)
Halon 1301 (bromotrzfluoromethane) is the safestfire extinguishing agent listedfrom the
standpoints ofboth toxicity and corrosion hazards.

What is the principle of operation of the continuous-loop fire detector system sensor?
Core resistance material which prevents current flow at normal temperatures.
The core material that serves as an insulator in a continuous-loopfire detection system
prevents currentflowing between the conductors under normal operating conditions. But,
when afire occurs, the loop is heated and the resistance ofthe insulating material becomes
low enough to allow current to pass between the two conductors and initiate a fire warning.

The most satisfactory extinguishing agent for an intake fire is


carbon dioxide.
Carbon dioxide, when properly used on an induction systemfire, will put out thefire and not
damage the engine as methyl bromide or dry chemicals would. CO2 does not produce the
lethalphosgene gas produced by carbon tetrachloride.

The explosive cartridge in the discharge valve of a fire-extinguisher container is


a life-dated unit.
The service life ofafireextinguzsher discharge cartridge is recommended by the
manufacturer, usually in terms ofhours. Some cartridges have a service life of5, 000 hours.

Why does the Fenwal fire detection system use spot detectors wired in parallel between two
separate circuits?
A short may exist in either circuit without causing a false fire warning.
A fire warning is initiated when the Fenwal spot detectors complete the circuit between the
two loops ofwire. A short circuit to ground in either loop will not produce afire warning.

Which of the following fire detection systems measures temperature rise compared to a reference
temperature?
Thennocouple
The thermocouple system uses active thermocouples in thefire area and a reference
thermocouple (referencejunction) enclosed in a dead air space between two blocks of
insulation material. The thermocouple system senses an excessive rate oftemperature rise to
indicate the presence ofafire.

The pulling out (or down) of an illuminated fire handle in a typical large jet aircraft fire
protection system commonly accomplishes what events?
Closes fuel shutoff, closes hydraulic shutoff} disconnects the generator field, and arms the fire-
extinguishing system.
When the illuminatedfire handle on ajet aircraft is pulled out or down, it closes thefiiel
shutofif closes the hydraulicfluid shutojf disconnects the generatorfield, and arms thefire-
extinguishing system. It does not discharge thefire-extinguishing agent.

A fire detection system operates on the principle of a buildup of gas pressure within a tube
proportional to temperature. Which of the following systems does this statement define?
Lindberg continuous-element system.
The Lindberg continuous-elementfire detection system operates on a buildup ofgas pressure
from gas released by the element inside the tube when it is heated. Increased gas pressure
moves a diaphragm and closes a switch to signal afire or overheat condition.

The fire detection system that uses a single wire surrounded by a continuous string of ceramic
beads in a tube is the
Fenwal system.
The Fenwal continuous-loop fire detection system uses a string ofceramic beads in a tube to
hold the centre conductor insulatedfi'om the outer tube. The conductivity ofthese beads
increases as the beads get hot.

TURBINE QUESTIONS 20
The fire detection system that uses two wires imbedded in a ceramic core within a tube is the
Kidde system.
The Kidde continuous-loopfire detection system uses two wires embedded in a ceramic
insulator whose conductivity increases as it gets hot. The insulator and the two wires are
housed in a continuous metal tube.

A fire detection system that operates on the rate-of-temperature rise is a


thermocouple system
A thermocouplefire detection system operates on the rate-of—temperature-rise. Active
thermocouples are located in thefire zone. A reference thermocouple is enclosed in a dead-air
space between two blocks ofinsulating material. As long as the active and reference
thermocouples are the same temperature, the system is inactive but in the event ofafire, the
temperature ofthe active thermocouple rises muchfaster than that ofthe reference
thermocouple so, the system signals afire.

Two continuous-loop fire detection systems that will not test due to a broken detector element are
the
Kidde system and the Fenwal system.
The Kidde and the Fenwalfire detection systems are the two continuous-loop fire detection
systems that will not test good ifthe detector element is broken. Both systems, however, will
detect afire even though they have a broken wire and test as being not good

In a fixed fire-extinguishing system, there are two small lines running from the system and
exiting overboard. These line exit ports are covered with a blowout type indicator disc. Which of
the following statements is true?
When the yellow indicator disc is missing, it indicates the fire-extinguishing system has been normally
discharged.
The yellow blowout indicator being blown out indicates that thefire extinguisher has been
discharged normally. Ifthe red indicator had been blown out, it would indicate that the
extinguisher has been discharged by an overheat (thermal) condition.

The most satisfactory extinguishing agent for an electrical fire is


carbon dioxide.
Carbon dioxide is the best ofthefire-extinguishing agents listed herefor extinguishing a fire
when there are energized electrical wires in thefire.

Which of the following fire detection systems will detect a fire when an element is inoperative but
will not test when the test circuit is energized?
The Kidde system and the Fenwal system
The Kidde and the Fenwalfire detection systems are the two continuous~loopfire detection
systems. Neither ofthese systems will test (show that they are good) ifthe detector wire is
broken. Both systems, however, will detect afire even though they have a broken wire.

Which of the following fire detection systems uses heat in the normal testing of the system?
The thermocouple system and the Lindberg system.
The thermocouple system and the Lindbergfire detection system both use heat to produce a
test signal. The other systems test by measuring continuity ofthe heat-sensitive element.

After a fire is extinguished, or overheat condition removed in aircraft equipped with a Systron~
Donner fire detector, the detection system
automatically resets.
When afire that has been detected by a Systron-Donnerpneumaticfire detection unit is
extinguished and the temperature drops, the hydrogen gas that had been released inside the
detection unit is re-absorbed and the pressure in the tube is reduced. This opens the
pneumatic switch and the system returns to normal, ready to signal anotherfire or overheat
condition.

A fuel or oil fire is defined as a


class B fro.
A class~Bfire is afire in which liquidfuel is involved.

TURBlNE ouesnous Q 21
A fire involving energized electrical equipment is defined as a
class C fire.
A class-Afire involves ordinary combustible materials such as wood, cloth, paper, upholstery
materials, etc.
A class-Bfire involvesflammablepetroleum products or otherflammable or combustible
liquids.
A class~Cfire involves energized electrical equipment.
A class-Dfire is afire in aflammable metal.

The use of water on class D fires


will cause the fire to burn more violently and can cause explosions.
A class~Dfire is afire in aflammable metal. Yhis type offire requires special handling. Water
used on a magnesiumfire will accelerate the burning and can cause an explosion. Special dry
powders are availablefor use whenever metalfires are possible.

For fire detection and extinguishing purposes, aircraft powerplant areas are divided into fire
zones based on
the volume and smoothness of the airflow through engine compartments.
Forfire detection and extinguishingpurposes, aircrafl powelplant areas are divided into fire
zones based on the volume and the smoothness ofthe airflow.

What will be the result of operating an engine in extremely high temperatures using a lubricant
recommended by the manufacturer for a much lower temperature?
The oil pressure will be lower than normal.
Ifan oil recommendedfor low-temperature operation is used in an engine when it is operating
in high—temperature conditions, the oil will have a lower viscosity than should be used in the
engine. Therefore, the oilpressure will be lower than recommended.

(1) Gas turbine and reciprocating engine oils can be mixed or used interchangeably.
(2) Most gas turbine engine oils are synthetic. J’*‘*‘\
Regarding the above statements,
only No. 2 is true.
Statement (1) is not true. The lubricating requirements ofa turbine engine are quite dzfierent
from those ofa reciprocating engine. Therefore, the types ofoil used in the two engines are
diflerent. Statement (2) is true because most ofthe lubricating oils used in turbine engines
have a synthetic base.

The time in seconds required for exactly 60 cubic centimetres of oil to flow through an accurately
calibrated orifice at a specific temperature is recorded as a measurement of the oil's
viscosity.
The method ofdetermining the viscosity ofan oil described here is the procedure used tofind
the Seconds Saybolt Universal (SSU) viscosity ofthe oil. The SSU viscosity is usually
converted into SAE viscosity which is morefamiliar to AP mechanics.

Upon what quality or characteristic of a lubricating oil is its viscosity index based?
Its rate of change in viscosity with temperature change.
The viscosity index ofan oil is a measure ofthe rate ofchange ofthe viscosity ofthe oil with a
change in its temperature.

Lubricating oils with high viscosity index ratings are oils


in which the viscosity does not vary much with temperature change.
The viscosity index ofan oil is a measure ofthe rate ofchange ofthe viscosity with a change
in its temperature. The higher the viscosity index ofan oil the less the viscosity changes as its
temperature changes.

Why are synthetic lubricants used in high-performance turbine engines?


They have less tendency to produce lacquer or coke and less tendency to evaporate at high
temperatures.

TURBINE QUESTIONS G 22
Synthetic lubricants are used in high-performance turbine engines because they have less
tendency to produce lacquer or coke and they have less tendency to evaporate at the high
temperatures they encounter. V

If all other requirements can be met, what type of oil should be used to achieve theoretically
perfect engine lubrication?
The thinnest oil that will stay in place and maintain a reasonable film strength.
An ideal engine oil is one with a low viscosity (it is thin andpours easily). The low viscosity
allows it to circulate easily within the engine. It must also maintain a reasonablefilm strength
so the lubricatingfilm will not break down at the high operating pressures and temperatures
encountered within the engine.

In addition to lubricating (reducing friction between moving parts), engine oil performs what
functions?
1. Cools.
2. Seals.
3. Cleans.
4. Prevents corrosion.
5. Cushions impact (shock) loads.
l, 2, 3, 4, 5.
In addition to reducingfriction between movingparts, an engine lubricant accomplishes all
five ofthe items listed in the alternativesfiir this question. It cools, seals, cleans, prevents
corrosion, and serves as a cushion between parts where impact loads are involved.

The type of lubricating oil that is used in a turbine aircraft engine is


synthetic.
Almost all turbine engines use a synthetic-base lubricating oil.

Which of these characteristics is desirable in turbine engine oil?


High flash point.
An engine lubricating oil should have a highflash point so that the vapours it gives ofcan
withstand the high temperatures encountered within the engine. The other alternatives listed
with this question are characteristics an oil should not have. (High pour point, Lowflash
point, High volatility.)

The viscosity of a liquid is a measure of its


resistance to flow.
The viscosity ofan oil is a measure ofits ability to flow at a specific temperature. This can be
thought ofas its resistance to flow.

What type of oil system is usually found on turbojet engines?


Dry sump, pressure, and spray.
Most turbojet engines use a dry-sump lubrication system.
A pressure pump directs the oil to the oiljets or nozzles located in the pressure lines ad/'acent
to or within the bearing compartments and the rotor shaft couplings. The oil is delivered to
the bearings in theform ofan atomized spray.

The engine's lubricating oil aids in reducing friction, cushioning shock, and
cooling the engine.
The lubricating oil in an engine primarily reduces friction, cushions shock and removes heat
from the engine.

Which of the following factors helps determine the proper grade of oil to use in a particular
engine?
Operating speeds of bearings.
The operating load, the rotational speed (the operating speed ofthe bearing) and the
operating temperatures are the most important considerations in determining the proper
grade ofoil to use in an aircraft engine.

Specific gravity is a comparison of the weight of a substance to the weight of an equal volume of

TURBINE QUESTlONS él 23
distilled water at a specific temperature. ‘
Specific gravity is the ratio ofthe weight ofa definite volume ofthe material being measured
to an equal volume ofpure water (distilled water) at its maximum density (at 4 °C).

Which of the following has the greatest effect on the viscosity of lubricating oil?
Temperature.
Temperature has a greater ejj‘ect than any ofthe other choices given here on the viscosity ofa
lubricating oil.

What advantage do mineral base lubricants have over vegetable oil base lubricants when used in
aircraft engines?
Chemical stability.
Mineral-based lubricating oils have a much greater chemical stability than do vegetable-
based lubricants.

Lubricants may be classified according to their origin. Satisfactory aircraft engine lubricants are
mineral or synthetic based.
The most satisfactory lubricantsfor aircraft engines have either a mineral base or a synthetic
base.

High tooth pressures and high rubbing velocities, such as occur with spur-type gears, require the
use of
an EP lubricant.
Some spur gears, and hypoid-type gearing having high tooth pressures and high rubbing
velocities, require the use ofextreme—pressure (EP) lubricants.

Manufacturers normally require turbine engine oil servicing within a short time after engine
shutdown primarily in order to
prevent overservicing _
Turbine engine manufacturers normalbr recommend engine servicing to be accomplished as
soon as practical afler engine shutdown to prevent overservicing. Overservicing may occur
because, in some engines, the oilfrom the storage tank seeps into the lowerportions ofthe
engine after the engine sitsfor some time without being operated.

The type of oil pumps most commonly used on turbine engines are classified as
positive displacement.
The oilpumps used in turbojet engine lubrication systems are ofthe positive displacement
type.

What will happen to the return oil if the oil line between the scavenger pump and the oil cooler
separates?
The return oil will be pumped overboard.
The scavenger pump pumps the oilfrom the engine through the cooler to the oil tank. Ifthe
line between the scavengerpump and the oil cooler separates, the scavenger pump willpump
all ofthe return oil overboard.

1) Fuel may be used to cool oil in gas turbine engines.


(2) Ram air may be used to cool oil in gas turbine engines.
Regarding the above statements,
both N0. 1 and No. 2 are true.
Statement (]).is true. Fuel may be used to cool the engine oil in an oil-to-fuel heat exchanger.
Statement (2) is also true. Ram air may be used to cool the engine oil in an air-to-oil heat
exchanger.

The main bearing oil damper compartments utilized in some turbine engines are used primarily
to
provide an oil film between the outer race and the bearing housing in order to reduce vibration
tendencies in the rotor system.

TURBINE QUESTIONS Q 24
A main bearing oil damper compartmentprovides an oilfilm between the outer race and the
bearing housing to reduce vibration tendencies in the rotor system and to allowfor a slight
misalignment.

What is the purpose of the last chance oil filters?


To filter the oil immediately before it enters the main bearings.
The last-chance oilfilters are used inside a turbine engine to filter the oil immediately before
it enters the main bearings.

In a jet engine which uses a fuel-oil heat exchanger, the oil temperature is controlled by a
thermostatic valve that regulates the flow of
oil through the heat exchanger.
The oil temperature in a turbojet-engine lubricating system is controlled by a thermostatic
valve that regulates theflow ofoil through the heat exchanger. The oil may bypass the heat
exchanger ifno cooling is needed.

In an axial-flow turbine engine, compressor bleed-air is sometimes used to aid in cooling the
turbine, vanes, blades, and bearings.
l Compressor bleea7—air, takenfiom an axial-flow turbine engine, is passed through hollow
guide vanes and turbine blades to help cool them. It z's also used to cool the bearings.

Oil picks up the most heat from which of the following turbine engine components?
Turbine bearing.
In a turbojet engine, the oilpicks the most heatfrom the turbine bearings.

Which of the following is a function of the fuel-oil heat exchanger on a turbojet engine?
Increases fuel temperature.
The oil—to~juel heat exchanger used with a turbojet engine, not only decreases the temperature
ofthe oil, but also increases the temperature ofthefuel to preventfuel icing.

Why are fixed orifice nozzles used in the lubrication system of gas turbine engines?
To provide a relatively constant oil flow to the main bearings at all engine speeds.
Pressurized oil is distributed to the engine main bearings in a turbojet engine throughfixed
orifice nozzles that provide a relatively constant oilflow at all engine operating speeds.

Possible failure related ferrous-metal particles in turbine engine oil cause an (electrical)
indicating-type magnetic chip detector to indicate their presence by
bridging the gap between the detector centre (positive) electrode and the ground electrode.
A magnetic chip detector warns ofpossible impending enginefizilure by indicating the
presence offerrous metalparticles. Ifthese particles are present in the oil, they will bridge the
gap in the detector between the centre electrode and ground electrode, completing an
electrical circuit to illuminate a warning light in the cockpit.

What would be the probable result if the oil system pressure relief valve should stick in the open
position on a turbine engine?
Insufficient lubrication.
Kthe oil-pressure reliefvalve in a turbojet engine stuck in the open position, oil wouldflow
through it rather than going through the passages in the lubrication system. This would result
in insufiicient lubrication ofthe engine.

What is the primary purpose of the oil-to-fuel heat exchanger?


Cool the oil.
The primary purpose ofthe 0il~to-fuel heat exchanger is to remove heatfi"om the engine
lubricating oil (to cool the oil). A secondary purpose ofthe oil~to-fuel heat exchanger is to
raise the temperature ofthefuel to prevent water in thefuelfrom precipitating out and
fleezing on thefuelfilters.

What unit in an aircraft engine lubrication system is adjusted to maintain the desired system
pressure?
Oil pressure relief valve.

TURBINE QUESTIONS Q 25
The oil-pressure reliefvalve in the lubrication system ofan aircraft engine is adjusted to
maintain the desired system pressure. Pressure above thatfor which the reliefvalve is set, lifls
the valvefrom its seat, and bypasses the excess pressure back to the inlet side ofthe oil pump.

Low oil pressure can be detrimental to the internal engine components. However, high oil
pressure
should be limited to the engine manufacturer's recommendations.
Oil pressure above that recommended by the engine manufacturer should be avoided because
excessive oilpressure can damage oil coolers and burst oil lines. A burst oil line could cause
a loss ofall ofthe lubricating oil.

What is the primary purpose of the oil breather pressurization system that is used on turbine
engines?
Provides a proper oil spray pattern from the main bearing oil jets.
The primary purpose ofthe breather pressurization system in a turbine engine is to insure a
proper oil spray pattern from the main bearing oiljets and tofurnish a pressure head to the
scavenge system.

The purpose of directing bleed air to the bearings on turbine engines is to


aid in removing heat from the bearings.
Directing compressor bleed-air to the bearings on turbine engines aids in removing heatfrom
the bearings. A considerable amount ofthe heat is removed by the cooling air rather than
being absorbed by the oil.

In order to relieve excessive pump pressure in an engine's internal oil system, most engines are
equipped with a
relief valve
Almost all aircraft engines have a pressure pump whose output exceeds the demands ofthe
engine. A reliefvalve is incorporated in the pressure portion ofthe system to relieve excess
pressure back to the inlet ofthe pump.

How are the teeth of the gears in the accessory section of an engine normally lubricated?
By splashed or sprayed oil.
The gear teeth in the accessory section ofan engine are lubricated by oil that is sprayedfi'om
the accessory shafl bearings and splashed around inside the accessory case.

What is the purpose of the check valve generally used in a dry sump lubrication system?
To prevent the oil from the supply ’ank from seeping into the crankcase during inoperative periods.
The check valve in the dry-sump lubrication system ofa reciprocating engine prevents oil
from the tankfrom seeping into the crankcase during the times when the engine is not
operating.

From the following, identify the factor that has the least effect on the oil consumption of a specific
engine.
Mechanical efficiency.
The mechanical efliciency ofan engine (the ratio ofthe power delivered to the propeller shaft
to the powerproduced in the engine) has the least e_)j’ect on the oil consumption ofany ofthe
other items listed in this question. The engine RPM, the characteristics ofthe lubricant and
the engine temperature all definitely affect the oil consumption ofan aircrafl engine.

Where is the oil temperature bulb located on a dry sump reciprocating engine?
Oil inlet line.
The oil-temperature bulb is located in the line between the oil tank and the inlet ofthe
pressure pump on a dry—sump reciprocating engine. Oil temperature measured at this point is
oil-inlet temperature. 7

If a full-flow oil filter is used on an aircraft engine, and the filter becomes clogged, the
bypass valve will open and the oil pump will supply unfiltered oil to the engine.
In case afull-flow oilfilter clogs, provisions are made (usually by a bypass valve) to provide
oil to the bearings. Q’thefilter clogs, the bypass valve will open and allow the pump to supply

TURBINE QUESTIONS él 26
the engine with unfiltered oil. It is much better to have unfiltered oil going through the engine
than for the engine to have no oil.

What is the primary purpose changing aircraft engine lubricating oil at predetermined periods?
The oil becomes contaminated with moisture, acids, and finely divided suspended solid particles.
Oil used in an aircrafl engine becomes contaminated with gasoline, moisture, acids, dirt,
carbon and metallic particles. Because ofthe accumulation ofthese harmful substances, the
oil used in an engine should be periodically drained and replaced withfresh oil.

What determines the minimum particle size which will be excluded or filtered by a cuno-type
(stacked disc, edge filtration) filter?
The spacer thickness.
A Cunofilter separates contaminants from the oil bypassing the oil between the disks in a
stack. The thickness ofthe spacers between the disks determines the separation ofthe disks.
This separation determines the minimum particle size that can befiltered out ofthe oil.

Why are all oil tanks equipped with vent lines?


To prevent pressure buildup in the tank. T
Oil tanks used in dry-sump lubrication systems ofaircraft reciprocating engines are vented to
the engine. The engine crankcase is then vented to the outside air. This venting system
prevents the heating and expansion ofthe oil causing a buildup ofair pressure inside the oil
tank.

(1) Wet sump oil systems are most commonly used in gas turbine engine.
(2) Oil in gas turbine engines is not diluted during cold weather.
Regarding the above statements,
only N0. 2 is true.
Statement (1) is not true. Wet-sump lubrication systems are not commonly used on gas turbine
engines. Statement (2) is true. The oil in gas-turbine-engine lubricating systems is not diluted
during cold weather operation.
_:r’\_ ‘E
ii“
The pumping capacity of the scavenger pump in a dry sump aircraft engines lubrication system
is greater than the capacity of the oil supply pump.
The scavengerpump in a dry-sump aircraft-engine lubricating system has a greater capacity
than the pressure pump. The oil returned to the tank by the scavenger pump has a greater
volume than the oil moved by the pressure pump because it is hotter and has air trapped in it.

In which of the following situations will the oil cooler automatic bypass valve be open the greatest
amount?
Engine oil below normal operating temperature.
When the engine oil is below its normal operating temperature, the automatic bypass valve is
wide open. This wide-open valve allows the oil to completely bypass the cooler. As the oil
warms up, the automatic bypass valve closes and directs the oil through the core ofthe cooler
so it can be cooled.

A turbine engine dry sump lubrication system of the self-contained, high-pressure design
consists of pressure, breather, and scavenge subsystems.
A turbine-engine dry~surnp lubrication system ofthe self-contained, high-pressure type, such
as is used on the Pratt & Whitney JT-3D engine, consists ofthree basic subsystems: The
pressure subsystem supplies oil to the main engine bearings and the accessory drives, the
scavenger subsystem scavenges oilfiom the bearing compartments and the accessory drives,
and the breather subsystem interconnects the individual bearing compartments and the
lubricating oil tank

Lube system last chance filters in turbine engines are usually cleaned
dining overhaul.
Last-chance oilfilters are installed in the oil lines to prevent plugging of the oiljets. Last-
chancefilters are accessible only during engine overhaul.

TURBINE QUESTlONS 6 27
The vent line connecting the oil supply tank and the engine in some dry sump engine installations
permits
the oil tank to be vented through the normal engine vent. .
Oil supply tanks used with dry~sump engines are normally vented to the engine crankcase. The
crankcase is, in turn, vented to the outside air. This method ofventing provides adequate
ventilation ofthe tank andprevents the loss ofoil through the vents.

An engine lubrication system pressure relief valve is usually located between the
pump and the internal oil system.
The pressure reliefvalve in an engine lubrication system is located between the pump and the
internal oil system to maintain the pressure ofthe oil being moved by the pump at the pressure
requiredfor the operation ofthe engine. Flow caused by excess pressure is dumped back into
the inlet ofthe pump.

Most turbine engine oil tanks incorporate a check relief valve in the tank venting system. The
purpose of this valve is to
prevent oil pump cavitation by maintaining a constant pressure on the oil pump inlet.
The check reliefvalve in the oil~tank vent system ofa turbojet engine prevents the oil pump
fiom cavitating by maintaining a constantpressure on the oilpump inlet.

If the oil in the oil cooler core and annular jacket becomes congealed, what unit prevents damage
to the cooler?
Surge protection valve. V
Ifoil becomes congealed in the core ofa cooler because ofits low temperature, the surge
protection valve in the cooler will open and allow the oil to flow around the core ofthe cooler
through an annularjacket. The operating engine oilflowing through thisjacket will warm up
the oil in the core so it will liquzfil and return to the lubrication system.

A rise in oil temperature and a drop in oil pressure may be caused by


foreign material under the relief valve.
ifforeign material becomes caught under the reliefvalve, it will hold the valve ofits seat and
cause the oilpressure to drop. Insufiicient oil pressure will cause inadequate lubrication. This
will cause an increase in the oil temperature.

The main oil filters strain the oil at which point in the system?
Just as it leaves the pressure pump.
The main oilfilter in an aircraft engine strains the oiljust afler it leaves the pressure pump.

Which type valve prevents oil from entering the main accessory case when the engine is not
running?
Check.
A check valve in an engine lubricating system prevents oilfrom the tankfrom entering the
accessory case when the engine is not running.

An oil tank having a capacity of S gallons must have an expansion space of


2 quarts.
An oil tank must have an expansion space ofnot less than 10% ofthe tank capacity, or one-
halfgallon. The required expansion spacefor a five-gallon oil tank is one-halfgallon, which
is two quarts.

As a general rule, a small amount of small fuzzy particles or grey metallic paste on a turbine
engine magnetic chip detector
is considered to be the result of normal Wear.
Smallfuzzy particles ofgrey metallic paste on the chip detector is considered satisfactory, and
is the result ofnormal wear. But, metallic chips orflakes are an indication ofserious internal
wear or breakage.

Why is expansion space required in an engine oil supply tank?


For oil enlargement and collection of foam.

TURBINE QUESTIONS G 2s
An expansion space is required in an engine oil tank because the oil expands (enlarges) when
it gets hot and when it collectsfoam.

The air and oil are separated in a jet engine oil system by returning the scavenged oil to
a de-aerator at the top of the reservoir.
Some oil tanks used with turbine engines incorporate de-aerator trays to separate the airfiom
the oil being returned to the tank by the engine scavenger system.

Which of the following bearing types must be continuously lubricated by pressure oil?
Plain.
Friction-type bearings, which are usually calledplain bearings, must be continuously
lubricated by pressure oil.

How does the ignition system of a gas turbine engine differ from that of a reciprocating engine?
A high-energy spark is required for ignition.
Turbine engine ignition systems are required to operatefor starting only, but they must
provide a high energy discharge at the igniter. An extremely hot spark is needed to relight the
engine ifitflames out at altitude.

In a turbine engine dc capacitor discharge ignition system, where are the high-voltage pulses
formed?
At the triggering transformer.
In the DC capacitor-discharge ignition system, such as is used on some turbojet engines, the
storage capacitor is charged to a given voltage. When this voltage is reached, the gas in the
discharge tube ionizes causing a portion ofthe accumulated charge toflow through the
primary ofthe triggering transformer. This induces a very high voltage in the secondary
winding connected to the igniter. This voltage is high enough tojump the gap andproduce the
trigger spark, which ionizes the gas in the gap. The storage capacitor discharges the
remainder ofits accumulated energy through the ionized gap together with the chargefrom
the trigger capacitor.

Why are turbine engine igniters less susceptible to fouling than reciprocating engine spark plugs?
The high-intensity spark cleans the igniter with heat.
Turbine-engine igniters are less susceptible to fouling than reciprocating~engine spark plugs
because the high-energy sparkjumping the gap cleans the contaminants out ofthe gap with
heat

The constrained-gap igniter plug used in some gas turbine engines operates at a cooler
temperature because 7
the construction is such that the spark occurs beyond the face of the combustion chamber liner.
The constrained-gap igniterplug used in some gas turbine engines operates at a cooler
temperature than other types ofigniters because it does not project into the combustion
chamber liner. The spark does not remain close to the plug, but arcs beyond theface ofthe
combustion chamber liner.

Generally, when removing a turbine engine igniter plug, in order to eliminate the possibility of
the technician receiving a lethal shock, the ignition switch is turned off and
the igniter lead is disconnected fiom the plug and the centre electrode grounded to the engine after
disconnecting the transformer- exciter input lead and waiting the prescribed time.
When removing an igniterplug, disconnect the transformer input lead, wait the length oftime
recommended by the manufizcturer, normally one to five minutes, then disconnect the igniter
lead and ground the centre electrode to the engine. This discharges the capacitor and makes it
safe to remove the igniterplug.

Great caution should be exercised in handling damaged hermetically sealed turbine engine
igniter transformer units because
some contain radioactive material.
Some transformer units contain radioactive caesium, and should be handled with extreme
care ifthe hermetic sealing has been damaged.

TURBlNE QUESTlONS 51 29
Igniter plugs used in turbine engines are subjected to high intensity spark discharges and yet
they have a long service life because they
do not require continuous operation.
The service life ofan igniter is much longer than the life ofa sparkplug used in a
reciprocating engine because the igniter does not operate continuously.

Which statement is correct regarding the ignition system of a turbine engine?


The system is normally de—energized as soon as the engine starts.
The ignition system ofa turbojet engine is operated only during the starting cycle. It is
normally de-energized when the engine is running.

The type of ignition system used on most turbine aircraft engines is


capacitor discharge.
The ignition system used on mostjet aircraft engines is the high-energy capacitor-discharge
type ofsystem.

Why do turbine engine ignition systems require high energy?


To ignite the fuel under conditions of high altitude and low temperatures.
Turbine-engine ignition systems require much higher energy than the ignition system for a
reciprocating engine. The reason is that turbine igniters must be able to consistently ignite the
fuel-air mixture under conditions ofhigh altitude and extremely low temperature.

Which of the following are included in a typical turbine engine ignition system?
1. Two exciter units.
2. One exciter unit.
3. Two transformers.
4. Two intermediate ignition leads.
5. Two low-tension leads. ’
6. Two high-tension leads.
7. One transformer.
1, 3, 4, 6.
.e“\.
A typical turbine-engine ignition system fires two igniters in combustors on the opposite sides
ofthe engine. In order to do this, two exciter units, two transformers, two intermediate
ignition leads and two high-tension leads are used.

When the water injection system on a turbine engine airplane contains water and is armed in the
cockpit,
the water injection valves are opened by a switch on their respective power levers in the cockpit.
After the water-injection system in a turbine engine is turned on with the cockpit switch, the
throttle is advanced to approximately the 86% RPMposition. At this position, a microswitch
in thefuel control is actuated. This switch supplies power to open the motor-operated shutofl"
valve that allows water to reach the engine-driven water pump where its pressure is boosted.
This water is discharged under a pressure ofabout 380 to 400 psi.

For what primary purpose is a turbine engine fuel control unit trimmed?
To obtain maximum thrust output when desired.
The adjustments allowed to be made on a turbine engine are the adjustment ofthe idling RPM
and the maximum RPM adjustment. These adjustments are commonly called trimming the
engine. An engine is trimmed to allow it to obtain maximum thrust output when it is desired.

Which type of fuel control is used on most of today's turbine engines?


Hydromechanical or electronic.
Most ofthe modernfitel controls used on turbine engines are ofthe hydromechanical or
electronic type.

Under which of the following conditions will the trimming of a turbine engine be most accurate?
No wind and low moisture.
To accurately trim a turbine engine, which is the adjustment ofthefuel control unit, there
should be no wind to create an untrue pressure at the engine intake. There should also be a

TURBINE QUESTlONS - 30
low moisture content in the air. This is because moisture, or water vapour, is much less dense
than standard (dry) air.

In turbine engines that utilize a pressurization and dump valve, the dump portion of the valve
drains the engine manifold lines to prevent fuel boiling and subsequent deposits in the lines as a result
of residual engine heat (at engine shutdown).
Thefitelpressurization and dump valve is usually required on engines that incorporate duplex
fuel nozzles to divide theflow into primary and main manifolds. At theflow requiredfor
starting andfor altitude idling, the pressurization valve is closed, and all ofthefuelpasses
through the primary line. The dump valve drains thefuel manifold to prevent thefuel boiling
as a result ofresidual engine heat. This boiling would leave solid deposits that could clog the
calibrated passageways.

What effect does high atmospheric humidity have on the operation of a jet engine?
Has little or no effect.
High atmospheric humidity, which afiects reciprocating engine power appreciably, has little
or no effect on a turbojet engine's thrust, fuelflow and RPM.

What are the positions of the pressurization valve and the dump valve in a jet engine fuel system
when the engine is shut down?
Pressurization valve closed, dump valve open.
When a turbojet engine is shut down, the pressurization valve is closed and the dumpvvalve is
open.

Which of the following is NOT an input parameter for a turbine engine fuel control unit?
Ambient humidity.
Jet enginefuel control units often monitor the engine or high-pressure compressor speed, the
compressor inletpressure and temperature, compressor discharge pressure, and burner can
pressure. Ambient humidity is not a controllingfactor.

(1) A supervisory electronic engine control (EEC) is a system that receives information
regarding various engine operating parameters and adjusts a standard hydromechanical fuel
control unit to obtain the most effective engine operation.
(2) A full-authority EEC is a system that receives all the necessary data for engine operation and
develops the commands to various actuators to control engine parameters.
Regarding the above statements,
both No. 1 and No. 2 are true.
Statement (I) is true. A supervisory EEC has a computer that receives information fiom the
various engine operating parameters and adjusts a standard hydromechanical FCU to obtain
the most effective engine operation. Statement (2) is also true. A full-authority EEC is a
system that receives all ofthe necessary datafor engine operation and develops the
commands to various actuators to control the engine parameters within the limits requiredfi>r
most eflicient and safie engine operations.

During what period does the fuel pump bypass valve open and remain open?
When the boost pump pressure is greater than fuel pump pressure.
The bypass valve around the vanes ofa vane-type engine-driven fuel pump opens and remains
open any time the boost pump pressure is greater than thefuelpump discharge pressure.

Which of the following statements concerning a centrifugal-type fuel boost pump located in a fuel
supply tank is NOT true?
The centrifugal-type pump is classified as a positive displacement pump.
A centrifugalpump is not a positive-displacementpump. A positive-displacementpump is one
that moves a constant volume offluid each time it revolves. A centrifugalpump can rotate
withoutpumping anyfuel ifits discharge is obstructed.

Where is the engine fuel shutoff valve usually located?


Aft of the firewall.
According to FAR 23.995, certificated aircraft must not have theirfiiel shutofi’valves located
on the engine side ofthefirewall.

’ TURBINE QUESTIONS Q 31
Boost pumps in a fuel system
provide a positive flow of fuel the engine pump.
Boostpumps in an aircrafifuel tank are primarily used to provide a positiveflow offuel to the
enginefuelpump. Boost pumps are also used to transferfitel, but this is not their primary
function.

What is the purpose of an engine-driven fuel pump bypass valve?


To prevent a damaged or inoperative pump from blocking the fuel flow of another pump in series with
it.
Almost all vane-type engine-drivenfuel pumps have a pump bypass valve through which fuel
canflow when the pump is not operating. This bypass valve allowsfuelflom the booster pump
to reach the enginefor starting and in case the engine-driven pump fails.

The fuel pump relief valve directs excess fuel to the


inlet side of the fuel pump.
Aircrafl-enginefuelpumps have a built-in reliefvalve that directs the excessfuel back to the
inlet side ofthe pump.

The primary condition that allows micro-organisms to grow in fuel is


the presence of water.
Micro-organisms ofvarious types can grow in an aircrafifuel tank and in storage tanks if
water is present. The micro-organisms live in the water andfeed on the hydrocarbon fuel.

It is desirable that fuel lines have a continuous slope upward or downward and NOT have sharp
curves or sharp rises and/or falls in order to
prevent vapour lock.
Fuel lines should have suflicient size to carry the maximum requiredfuelflow under all
operating conditions, and should have no sharp curves or sharp rises and/orfalls which
would accumulate vapours and cause vapour lock. Fuel lines should also be kept awayfrom
hot parts ofthe engine and exhaust system.
€&

The fuel systems of aircraft certificated in the standard classification must include which of the
following? '
A positive means of shutting off the fiiel to all engines.
All aircraftfuel systems must have some positive means ofshutting thefuel ofi"to all ofthe
engines.

Where should the main fuel strainer be located in the aircraft fuel system?
At the lowest point in the fuel system
The mainfitel strainer in an aircraft should be located at the lowest point in thefuel system so
it can collectforeign matterfi'om the line between the tank and the engine and where it can
also serve as a water trap.

Where physical separation of the fuel lines from electrical wiring or conduit is impracticable,
locate the fuel line
below the wiring and clamp the line securely to the airframe structure.
Kfuel lines and electrical wire bundles must be routed through the same compartment, the
fuel line must be below the wiring to preventfuelfrom dripping into the wiring ifthefuel line
should leak. The wire bundle must be securelyfastened to the airfiame structure, never to the
fuel line.

What is a characteristic of a centrifugal-type fuel boost pump?


It separates air and vapour from the fuel.
A centrzfugal—type boostpump, located in afuel tank, separates the air and vapourfrom the
fiiel before thefuel isfirrced out ofthe tank through thefitel line. The vapours rise to the top of
the tank.

Fuel boost pumps are operated


to provide a positive flow of fuel to the engine.

TURBINE QUESTIONS él 32
Fuel boostpumps are operated to provide a positiveflow offuel to the enginefor starting and
for high-altitude operation when there is a possibility ofvapour lock. They are also used in
many installations to transferfuel.

A pilot reports that the fuel pressure fluctuates and exceeds the upper limits whenever the
throttle is advanced. The most likely cause of the trouble is
a sticky fL1€l pump relief valve.
Fluctuatingfuelpressure andpressure that sometimes exceeds the upper limitsfor thefuel
pressure can be caused by a sticky reliefvalve in thefitel pump.

A fuel strainer or filter must be located between the


tank outlet and the fuel metering device.
According to FAR 23.997, there must be afuel strainer orfilter between thefiiel tank outlet
and the inlet ofeither thefiiel metering device or an engine-driven, positive-displacementfitel
pump, whichever is nearer thefizel tank outlet.

Fuel lines are kept away from sources of heat and sharp bends and steep rises are avoided to
reduce the possibility of
vapour lock.
A vapour lock is a condition in afitel line in which fuel vapours become trapped in a bend. If
the vapour pressure ofthe trappedfuel is higher than thefuelpressure, no fiiel canflow to the
carburettor. Sharp bends and steep rises should be avoided, andfuel lines should not be
routed in an area where thefil€l can absorb heat.

Fuel crossfeed systems are used in aircraft to


maintain aircrafi stability.
The main purpose ofafuel cross-feed system in a multi-engine aircraft is to allow any engine
to operatefrom anyfuel tank. In case one enginefails, the cross-feed system allows thefuel
from all ofthe tanks to be used evenly. This helps maintain aircrafl stability.

A fuel pressure relief valve is required on


engine-driven vane-type fuel pumps.
Vane-type engine-drivenfitelpumps are positive-displacement pumps and require
a pressure reliefvalve.

A rotary-vane pump is best described as


a positive-displacement pump.
A rotary vane-type pump is a positive-displacement pump.

Fuel pressure produced by the engine-driven fuel pump is adjusted by the


relief valve adjusting screw.
The pressure produced by a vane-type engine-driven pump is adjusted by the reliefvalve
adjusting screw.

Gasoline and kerosene are used as turbine engine fuels. How do they compare in heat energy?
Kerosene has more heat energy per gallon than gasoline.
Kerosene has more heat energy per gallon than gasoline, but it has less heat energy per
pound. Kerosene has a nominal heat energy content ofI8, 400 Btu per pound and it weighs 6. 7
pounds per gallon. This gives a heat energy content of123,280 Btu per gallon. Aviation
gasoline has a nominal heat energy content of.20, 000 Btu per pound and it weighs six pounds
per gallon. Gasoline has 120, 000 Btu ofheat energy per gallon.

What are the principle advantages of the duplex fuel nozzle used in most turbine engines?
Provides better atomization and uniform flow pattern.
A duplexfuel nozzle in a turbine engine provides better atomization and a more uniformflow
pattern through a wide range ofengine operation than a simplex nozzle.

It is necessary to control acceleration and deceleration rates in turbine engines in order to


prevent blowout or die~out.

TURBINE QUESTIONS Qt 33
Thefuel control in a turbine engine controls the amount offuel metered to the engine during
acceleration and deceleration. L’too muchfiiel is metered to the engine when acceleration is
wanted, thefuel will be discharged before the compressor can build up its speed. Thefire will
blow out, because there is too muchfiielfiar the amount ofavailable air. When the power
control lever is closed, callingfor a deceleration, thefitel control meters thefuel to prevent
there being too littlefuelfor the amount ofair the compressor is moving through the engine as
it slows down.

When trimming a turbine engine, the fuel control is adjusted to


set idle RPM and maximum speed or EPR.
Trimming a turbine engine means adjusting thefizel control unit to set the idle RPM and the
maximum speed or EPR ofthe engine.

Which of the following turbine fuel filters has the greatest filtering action?
Micron.
The three most common types offuelfilters used in turbine-enginefiiel systems are: the
micronfilter, the wafer screenfilter, and the plain screen mesh filter. The micronfilter has the
greatestfiltering action. It is capable offiltering outparticles down to about .70 microns.

What is the purpose of the flow divider in a turbine engine duplex fuel nozzle?
Creates the primary and secondary fuel supplies.
A flow divider used with a turbine-engine duplexfuel nozzle creates a primary and a
secondaryfi(€l supply which are discharged through separate concentric spray tips. This
action provides the proper spray anglefor allfuelflow rates.

What causes the fuel divider valve to open in a turbine engine duplex fuel nozzle?
Fuel pressure.
Theflow divider valve can be a self-contained unit or it can be built into each nozzle. Either
type is a spring-loaded valve set to open at a specificfuelpressure.

The component(s) in turbine engines that operate(s) at the highest temperatures is/are the
first stage turbine nozzle guide vanes.
The temperature inside a turbine engine is the highest as the gases pass through the nozzle
guide vanes andfirst stage turbine.

The air passing through the combustion chamber of a jet engine is


used to support combustion and to cool the engine.
Air passing through the combustion chambers ofa turbine engine is used to support
combustion as well as to cool the engine. Only about 25% ofthe airpassing through the
combustion chamber is actually involved in the combustion process.

Why is high nickel chromium steel used in many exhaust systems?


Corrosion resistance and low expansion coefficient.
High nickel-chromium steel is used in exhaust systems because ofits high corrosion resistance
and its low coefiicient ofexpansion.

The hot section of a turbine engine is particularly susceptible to which of the following kind of
damage?
Cracking.
Cracking is the chiefform ofdamagefound in the hot section ofa turbine engine. The reason
for the cracking is the extremes oftemperature and temperature changes that exist in the hot
section.

On turbine-powered airplanes, how much reverse thrust is usually required for minimum
braking requirements?
At least 50 percent of the full forward thrust of the engine.
To satzsjjr the minimum braking requirements after landing, a thrust reverser should be able to
produce, in reverse, at least 50% ofthefull-forward thrust ofwhich the engine is capable.

TURBINE QUESTIONS 34
Which of the following indicates that a combustion chamber of a jet engine is not operating
properly? '
Hot spots on the tail cone.
Hot spots on the tail cone ofa turbojet engine could indicate uneven burning within the
engine. This uneven burning could be caused by a malfunctioning combustion chamber.

What is the function of the thrust reverser of a turbine engine?


To reverse the flow of exhaust gases.
Thrust reversers are used on turbojet engines to reverse theflow ofexhaust gases and help
slow the aircraft.

How are combustion liner walls cooled in a gas turbine engine?


By secondary air flowing through the combustion chamber.
Combustion liner walls in a turbine engine are cooled by secondary airflowing through the
combustion chamber.

How is aircraft electrical power for propeller deicer systems transferred from the engine to the
propeller hub assembly?
By slip rings and brushes.
Electricalpower is transferredfrom the engine to the propellerfor propeller deicing by the
use ofslip rings and brushes.

How is anti-icing fluid ejected from the slinger ring on a propeller?


By centrifiigal force.
Anti-icingfluid is ejectedfi'om the slinger ring on a propeller by centrzfugalforce as the
propeller rotates.

Propeller fluid anti-icing systems generally use which of the following?


Isopropyl alcohol.
The anti-icingfluid used on a propeller is normally isopropyl alcohol.

What is a function of the automatic propeller synchronizing system on multi-‘engine aircraft?


To control engine RPM and reduce vibration.
Automatic propeller synchronizing systems on multi-engine aircraft hold the RPM ofall ofthe
engines the same to minimize vibration.

Ice formation on propellers, when the aircraft is in flight, will


decrease thrust and cause excessive vibration.
Iceformation on a propeller, when the aircrafl is in flight, will alter the airfoil shape ofthe
propeller blade. Altering the airfoil shape decreases the thrust and causes excessive vibration.

What type of imbalance will cause a two~blade propeller to have a persistent tendency to come to
rest in a horizontal position (with the blades parallel to the ground) while being checked on a
propeller balancing beam?
Vertical.
Ifa propeller comes to rest on a balance stand with its blades in the horizontal position, one
side ofthe propeller hub is heavier than the other. The propeller is said to be vertically
unbalanced.

What is the purpose of an arbor used in balancing a propeller?


To support the propeller on the balance knives.
When a propeller is to be balanced, it is placed on an arbor which supports it on
the balancing knife edges.

If a blade of a particular metal propeller is shortened because of damage to the tip, the remaining
blade(s) must be
reduced to confonn with the shortened blade.
Ifone blade ofa metalpropeller is shortened or its ‘shape is changed to dress out damage to
the tip, the other blades must be made to conform to the shortened blade.

TURBINE QUESTIONS Gf 35
Apparent engine roughness is often a result of propeller unbalance. The effect of an unbalanced
propeller will usually be
greater at high RPM.
The centrifugalforce caused by the rotation ofa propeller will cause‘ any unbalanced
condition to befelt more at high engine RPM than at low RPM

Propeller aerodynamic (thrust) unbalance can he largely eliminated by


correct blade contouring and angle setting.
Aerodynamic unbalance ofa propeller results when the thrust ofthe blades is unequal. This
type ofunbalance can be largely eliminated by checking blade contour and blade angle
setting.

A powerplant using a hydraulically controlled constant~speedpropeller is operating within the


propeller 's constant-speed range at afixed throttle setting. Ifthe tension ofthe propeller governor
control spring (speeder spring) is reduced by movement ofthe cockpitpropeller control, the propeller
blade angle will
increase, engine manifold pressure will increase, and engine RPM will decrease.
Kan engine is operated in the constant-speed range and the compression ofthe propeller
governor speeder spring is reduced, the centrifugalforce on theflyweights willproduce an
overspeed condition which will cause the propeller blade angle to increase. This increase in
blade angle will cause the engine RPM to decrease. h’the throttle position has not been
changed, the manifoldpressure will increase.

Why is the pulley stop screw on a propeller governor adjustable?


To limit the maximum engine speed during takeoff.
The pulley stop screw on a manually adjustable constant-speedpropeller governor adjusts the
maximum RPM the governor allows the engine to produce. This screw actually limits the
maximum amount the speeder spring is allowed to compress.

During engine operation at speeds lower than those for which the constant-speed propeller
control can govern in the INCREASE RPM position, the propeller will a>"$“-..,.
’>\; ~/'
hi

remain in the full LOW PITCH position.


During engine operations at speeds lower than those which the constant-speed governor can
control, the propeller acts as though it were afixed-pitch propeller in thefull LOWPITCH
position. Full low pitch produces the highest RPM.

When engine power is increased, the constant~speed propeller tries to function so that it will
maintain the RPM, increase the blade angle, and maintain a low angle of attack.
When engine power is increased in an engine equipped with a constant-speedpropeller, the
governor causes the blade angle to increase to hold the RPM constant. The increase in power
willpossibly increase the airspeed. This will hold the propeller angle ofattack low.

The propeller governor controls the


oil to and from the pitch changing mechanism.
A propeller governor controls the oilflowing to andfrom the pitch-changing mechanism ofa
hydraulically actuatedpropeller.

During the on-speed condition of a propeller, the


centrifugal force of the governor flyweights is equal to the speeder spring force.
During the on~speed condition ofa propeller governor, the centrifugalforce on the governor
flyweights is exactly balanced by theforce produced by the speeder spring.

What actuates the pilot valve in the governor of a constant-speed propeller?


Governor flyweights.
The pilot valve in a propeller governor is actuated by a balance offorces between those
produced by centrzfugalfinrce acting on theflyweights and the compression ofthe speeder
spring.

What action takes place when the cockpit control lever for a hydromatic, constant-speed
propeller is actuated?

TURBINE QUESTIONS é‘ 36
Compression of the speeder spring is changed.
When the cockpit controlfor a Hydromatic constant-speedpropeller is actuated, the
compression ofthe speeder spring is changed. To increase the RPM, the compression is
increased. This increased springfizrce requires a higher RPM to produce enough centrifugal
firrce on theflyweights to overcome it.

What will happen to the propeller blade angle and the engine RPM if the tension on the propeller
governor control spring (speeder spring) is increased? "
Blade angle will decrease and RPM will increase.
When the tension (actually the compression) on the propeller governor control speeder spring
is increased, the governor gives the indication ofan underspeed condition. The propeller
blade angle will decrease, allowing the RPM to increase.

How is the speed of a hydromatic constant-speed propeller changed in flight?


By changing the load tension against the flyweights in the governor.
The speed ofa Hydromatic constant-speedpropeller is changed by controlling the
compression flood tension) ofthe speeder spring inside the governor. The compression ofthe
speeder spring opposes the centrifugalfiarce on theflyweights in the governor. It is the
balance between the action oftheflyweights and the compression ofthe speeder spring that
determines the speed ofthe propeller.

When the centrifugal force acting on the propeller governor counterweights overcomes the
tension on the speeder spring, a propeller is in what speed condition?
Overspeed.
When the centrtfitgalforce acting on the propeller governor counterweights overcomes the
force ofthe speeder spring, the propeller is in an overspeed condition. The governor will
cause the propeller blade angle to increase to slow the propeller down.

What operational force causes the greatest stress on a propeller?


Centrifugal force.
The greatest stress on a propeller caused by an operationalforce is centrzfiigalforce
caused by the rotation ofthepropeller.

What operational force tends to increase propeller blade angle?


Aerodynamic twisting force.
Aerodynamic twistingforce (ATF) tends to rotate a propeller blade to its high-pitch blade
angle. This is opposite to the effect ofcentnfiigal twistingforce (CTF).

How is a propeller controlled in a large aircraft with a turboprop installation?


By the engine power lever.
The turbopropfuel control andpropeller governor are interconnected and operate in
coordination with each other. The engine power lever directs a signalfrom the cockpit to the
fuel controlfiir a specific amount ofpowerfrom the engine. Thefi{€l control and the propeller
governor together, establish the correct combination ofRPM, fitelflow andpropeller blade
angle to create sufiicientpropeller thrust to provide the desiredpower.

How does the aerodynamic twisting force affect operating propeller blades?
It tends to turn the blades to a high blade angle.
Aerodynamic twistingjbrce tends to turn the blades ofa propeller to their high-pitch blade
angle.

Which of the following best describes the blade movement of a hydromatic propeller that is in the
high RPM position when reversing action is begun?
Low pitch directly to reverse pitch.
A reversingpropeller operating in the high RPM (lowpitch) condition when the reversing
action is begun will go fiom low pitch directly to reverse pitch. Before this can happen, the
low pitch stop levers must be released to allow the propeller pitch to flatten out below its
normal low pitch position.

Propellers exposed to salt spray should be cleaned with

TURBINE QUESTIONS 37
fresh water.
Propellers that have been exposed to salt water should beflushed with flesh water until all
traces ofsalt have been removed. This should be accomplished as soon as possible after the
salt water has splashed on the propeller. Afterflushing all parts should be thoroughly dried
and all ofthe metal parts coated with clean engine oil or with a suitable equivalent. ‘

How can a steel propeller hub be tested for cracks?


By magnetic particle inspection.
A steelpropeller hub can be inspectedfor cracks by using a magnetic particle inspection
method.

Repairs of aluminium alloy adjustable pitch propellers are not permitted to be made on which of
the following propeller blade areas?
Shank.
No repairs are permitted to the shanks ofaluminium alloy aa§ustable~pitch propeller blades.

Which of the following functions requires the use of a propeller blade station?
Measuring blade angle.
The only one ofthese choices that requires the use ofthe propeller blade stations is that of
measuring the blade angle. Blade angle is always specified at a particular blade station.

The propeller blade angle is defined as the acute angle between the airfoil section chord line (at
the blade reference station) and which of the following?
The plane ofrotation.
The blade angle ofa propeller is defined as the acute angle between the chord ofa particular
blade section and the plane in which the propeller blades rotate (the plane ofrotation).

During which of the following conditions of flight will the blade pitch angle of a constant-speed
propeller be the greatest?
High-speed, high-altitude cruising flight. _
A constant-speedpropeller will have the highest blade angle during high-speed, high-altitude
cruisingflight. In all ofthe other alternatives with this question require the propeller to have
a low pitch angle.

If a hydromafic propeller is feathered and then immediately unfeathers itself, a probable cause of
the trouble is that the
pressure cutout switch is stuck in the closed position.
lfa Hydromatic propellerfizathers (ind then automatically and immediately unfeathers, it is an
indication that the pressure cutout switch in the governor is stuck in the closedposition and
does not open the circuit to shut ofi"thefeather pump motor. The pump continues to build up
pressure until it shifts the distributor valve and unfeathers the propeller.

The actual distance a propeller moves forward through the air during one revolution is known as
the
effective pitch.
The efirective pitch ofa propeller is the distance the propeller actually moves through the air
during one revolution.

The pitch-changing mechanism of the hydromatic propeller is lubricated by


the pitch-changing oil.
The dome ofa Hydromatic propeller isfilled with engine oil. This oilprovides all ofthe
lubrication neededfor the pitch changing mechanism.

Propeller blade stations are measured from the


hub centreline.
Propeller blade stations are measured in inchesfiom the centre line ofthe propeller hub.

The thrust produced by a rotating propeller is a result of


an area of decreased pressure immediately in front of the propeller blades.

TURBINE QUESTlONS G7 38
Thrust is produced by a rotating propeller in the same way lzfi is produced by a wing. An area
ofdecreasedpressure is produced immediately infiont ofthe propeller. Aerodynamicforces
cause the propeller to move into this area oflow pressure.

Why is a constant~speed counterweight propeller normally placed in full HIGH PITCH position
before the engine is stopped?
To prevent exposure and corrosion of the pitch changing mechanism.
The Hamilton Standard counterweightpropeller is usually put in the high-pitch position
before the engine is stopped. Thz's allows the cylinder to move back and cover the piston so it
will not be exposed to moisture and otherforces that could cause rust and corrosion. On
propellers in which this is not a problem, the engine should be shut down with the propeller in
low pitch, instead ofhigh pitch.

The low pitch stop on a constant-speed propeller is usually set so that


the engine will turn at its rated takeoff RPM at sea level when the throttle is opened to allowable
takeoff manifold pressure.
The low pitch stop on a constant-speedpropeller is set so the engine will turn at its rated
takeoffRPM at sea level when the throttle is opened, to get the allowable takeoflmanifold
pressure.

The angle~of-attack of a rotating propeller blade is measured between the blade chord or face
and which of the following?
Relative airstream.
The angle ofattack ofa propeller blade is the angle between the blade chord orface, and the
relative airstream.

The centrifugal twisting moment (CTM) of an operating propeller tends to


reduce the pitch angle.
The centrifugal twisting moment (CTM) acting on a propeller blade tends to rotate the blade
to a low-pitch angle.

Which of the following is identified as the cambered or curved side of a propeller blade,
corresponding to the upper surface of a wing airfoil section?
Blade back.
The curved side ofa propeller blade (the side that corresponds to the upper sutfizce ofa wing
airfoil section) is called the blade back. Theflatportion ofa propeller blade is called the
bladeface.

Which of the following best describes the blade movement of a full-feathering, constant—speed
propeller that is in the LOW RPM position when the feathering action is begun?
High pitch directly to feather position.
Kafull-featheringpropeller is in the low RPM, high pitch position, when thefeathering
action is begun, the propeller will go directlyfrom high pitch to thefeather position.

The holding coil on a hydromatic propeller feathering button switch holds a solenoid relay closed
that applies power to the propeller
feathering pump motor.
The holding coil on a Hydromatic propellerfeathering system holds thefeather button
depressed and thefeatheringpump operating until the propeller isfeathered. Then, the oil
pressure builds up high enough to open the oilpressure switch in the governor.

Blade angle is an angle formed by a line perpendicular to the crankshaft and a line formed by the
chord of the blade.
Propeller blade angle is measured in degrees and is the acute angle between the chord ofthe
blade and the plane ofrotation. The plane ofrotation is perpendicular to the cranlashafi‘.

TURBINE QUESTIONS Q1 39
ECQEEOO

U6
UO§®E_
QEUCQg3§_gmNE50“mgssw
_ M‘E‘_OZ
_fig
Em“
g_2:
$283m_
Bwgaeg
SEW
6?Cgggwgfi
____Wgmfi
OzOC_£_ _ _eB gfltn
U6
ggtE
Eve
Qgeog
OCOQgwga
wGCEQWQ_
E§®_wE®OE_ ®_nmQ®C_95
:O_ £g:>> 9&0
tgw
E
UCN
Ea
msmg
QCmC__%wQ>
Og§C_QLZ_GQ$wW9mgggo
n_59%?_N>3%
63>)
Egg
C_EM
LOW®QMgg__=__OC
N_>OE9QEgge
mW_DLQ®§ECgOwQ£O O:®_ EWCW
USN
UQWD
UCM
hm
E
WQOm>®Q
m_UO£®E
$23OH
’20%_‘
EN;
®__Oh_®#
_ Mw5@>_a GwC®QEOO E
$9
B
QCECQQEQI
_mQUDUOE
®>9®C®
2:
9%
25C_On$_tO_®Q>ON_En_mgwOEB KEBE
U6
UCN
QOEI0NCO
59%?W
ECOEEUQO
M$OQ5n_
mC@OlE_QE_E CGEQOEOQAW
E
9
EM
@559Q53
EOEE2_E‘
®_Qfigggg
8920
N_C_2®0‘5>w_$‘_Q_2gm
_ amm_QoQ‘oa%€3ME$moUmgm
imWQCECQ
QO_Comggg
$gogi E£55
megSEED
mgCOwi
mummogsm
_cmo nB
_oE gmzcmg p
/‘Kl\

_U£052)
_'U_H__W;
WWUOLOMGCMHOQ
$QQDUOLQ
QEGCQQCEMSHWNWNCM
H>_‘_OmH*_ _£
“O
=2395
BC
Ex
®G9__g
WEC203
QEVEEL2
OCQ®OtQQEEO_
EVEQO
$0
E
WC255
LO®_
QUEQ
MWQW
mafigg
255
O%O§_ NNOC VG
Egmgu
$.WSQSO
_>‘Wfim
~QCBLBH
:WQEQCQ
Q>>OQ
@®M®EEU
_ MQEOO
ace
mgvgs
2_E‘W¢'C_E\! _;_wCm mag
E6
COEEOO
2&3
LQEEOQO
80_>%
L65
%_ $®>9>OQ
is
ENb_g_%
was
m‘£55gm9Ou56£
Zggkfiw
N _OMO_mH_ E
V6
QUE
EEQAWCOECQgag
M
®_l_p_O®_n§_O
CO:LEEmég E2563
“O
_2%
OH
\OCawOEOn§_®Efl
®_ QEOU_2O&mbw:
E
63
UQWWQQ
:Ea
WmCg;
QXQEJU
W;w_JO§
t®20_Nm_O®_£O_g _u_O
E5
E
O8%
E3200
E303
9:
_®>_®>
M‘_EOEO®>Om BENUmgn
359$
®C55__Co$562
mg
950229
QWOQSHM
“O
xgw
UQMOB
UQQB
COwfimt
mN
L®>_$>
OWWQLstgw
QEOU w E
2:
Cggi
EQEEUr_O_aBEO mg
>_&_6___®QWEQ
vsW®ULNCO
Mg
gfifi
Omémg
mE CEMQ>I_EU<W
E
_Eva
®>$Ow
m_:m_EO9W®_w\QE%m_ZmQE8EOw>OUg OC®_OE® £Hgt
U_O
EEEE
BOSE
958%
>mgM
UgmgGEE; U6
O880$
>_$596QgggN
U_C$>fi_UE@Dzggg
B
HAgfi
Owfimgw
'2Cg
_ O®EC®__U®_NNOCmE__gOafigfim
E8mg
2:
CO9_$$3_
9____30a
EH5
®_NNOC V6
Etgh
®$82
WWME_“E
9_§>Oa
mC_§UBO:
UE_‘ W Om
h_O
EQHQAWCOECQ®m_
M
COENLQQO _M_\_O2U__OtOfiQMO
_%M__
_2E8
éECG?5&5
EmmC:26¢WWNE
_ aEOO 62
B§E8
E8&6
®E®EQ58
®O_Mm_tQWN_>UM

kc
E25CO_NC_£_>>
“Engmg
$596 §>m5 zgw
B
$O®%Ea
Lggg>__fi
OW EQEO :5_eEOU
E:NagsaENEME
Ea
B On_EEEE
53%
_OCOEEOO
'=EQWQAW
Emflwefihfla
_ (_OO E_gC8U®®Q2
mégm%a_O€B
M ®>O Hv_QO_$95WCO
_ObC8
gel
M_ p_Q®‘Op_3

ggm
@>_®>
mC_ggjCg@QOw
:Ogzggg
\WUOCE
_:< E

QOCGCQERE

You might also like